You are on page 1of 251

WORK AND ENERGY

A. Work and Work-Energy Theorem


B. Kinetic Energy
C. Potential Energy: Gravitational and Elastic
D. Principle of Conservation of Energy
Work and Work-Energy Theorem

 If a body moves through a displacement s while a constant force F


acts on it in the same direction, the work(W) done by the force
on the body is W = Fs. However, if the constant force is acting on
the body at an angle, the work done is W = Fs cos θ.
 The unit of work is Joule(J), where I J = 1 Nm. The other unit for
work is erg where 1 erg = 1 dyn cm (1 J = 1 X 107 ergs). The
English System of Measurement uses the unit ft.lb where 1 ft.lb =
1.356 J.
F
F θ

s s
WORK: A CONTINUATION

 Work can be expressed as a scalar product of constant force F and


displacement s where W = F•s or W = Fscos θ.
 Suppose a 100-N force acts on a 2-kg block at an angle 600 with
the horizontal. The block is displaced 4 meters along a rough
surface with a coefficient of kinetic friction μk = 0.3. Find the
F Fsin 600 (-j) a) work done by the gravitational force,
600 n b) work done by the normal force,
Fcos 600(î)
c) work done by the frictional force,
fk
w
4m d) work done by the force F, and
e) total work.
WORK: A SOLUTION

a) Wg = Fscos θ = mgs cos 900 = 0 Example: What is the work done by a


b) Wn = Fscos θ = ns cos 900 = 0 force F = 2N î + 4N ĵ – 3N ḱ acting
c) Wfk = Fscos θ = μkmgs cos 1800 parallel through a displacement
= 0.3(2 kg)(9.8 m/s2)(4m)(-1) s = 1m î + 2m ĵ – 1m ḱ.
= - 23.52 J W = F•s
d) WF = Fscos 600 = (100 N)(4 m)(0.5) = (2N î + 4N ĵ – 3N ḱ)•(1m î + 2m ĵ – 1m ḱ)
= 200 J = 2 Nm + 8 Nm + 3 Nm
e) WT = Wg + Wn + Wfk + WF = 13 Nm
= 0 + 0 – 23.52 J + 200 J W = 13 J
= 176. 48 J
POWER

Power, in everyday conversation, is synonymous with “energy” or


“force”. In physics, power is the time rate at which work is done or it
is the rate of doing work. Power is a scalar quantity.
Average power(Pav) is defined to be
Δ𝑊
Pav = ,
Δ𝑡
where ΔW = the work done during time interval
Δt = the duration of time interval

Δ𝑊 𝑑𝑊
Instantaneous Power(P) P = lim =
Δ𝑡→0 Δ𝑡 𝑑𝑡
UNITS OF POWER

 The SI unit of power is the watt(W). One watt equals 1 Joule per
second: 1 W = 1 J/s.
 Another common unit of power is the horsepower(hp).
1 hp = 746 W = 0.746 kW
The kilowatt-hour(kW.h) is the usual commercial unit of electrical
energy. One kilowatt-hour is the total work done in 1 hour(3600 s)
when the power is 1 kilowatt(103 J/s), so
1 kW.h = (103 J/s)(3600 s) = 3.6 X 106 J = 3.6 MJ
The kilowatt-hour is a unit of work or energy, not power.
POWER: AN EXAMPLE

 A 50-kg marathon runner runs up the stairs to the top of a 443-m


tower. To lift herself to the top in 15 minutes(900 s), what must be
her average power output? Express your answer in watts, in
kilowatts, and in horsepower.
 W = mgh = (50 kg)(9.8 m/s2)(443 m) = 2.17 X 105 J
2.17 𝑋 105 𝐽
 Pav = = 241 W = 0.241 kW = 0.323 hp
900 𝑠
443 𝑚
 Alternative solution: vav = = 0.492 m/s
900 𝑠
 Pav = FIIv = (mg)vav = (50 kg)(9.8 m/s2)(0.492 m/s)
 = 241 W = 0.241 kW = 0.323 hp
POWER: ANOTHER FORMULA

 Suppose that a force F acts on a body while it undergoes a


vector displacement Δs. If FII is the component of F
tangent to the path(parallel to Δs), then the work done by
the force is ΔW = FII Δs. The average power is
𝐹𝐼𝐼Δ𝑠 Δ𝑠
 Pav = = FIIΔ𝑡 = FIIvav
Δ𝑡
 Instantaneous power P is the limit of this expression as
 Δt → 0: P = FIIv . We can also express this equation in terms
of the scalar product: P=F•v.
Work and Power: Exercises

1. A person whose mass is 60 kg climbs the stairs up to the next floor


which is 6 meters above. (a) What is the work done against gravity?
(b) What is the work done by gravity? (c) What is the work done by
all the external forces? Ans: (a) 3.53 X 103 J (b) – 3.53 X 103 J (c) 0
2. A hoist powered by 20-hp motor is used to raise a 500-kg bucket of
concrete to a height of 80 m. If the efficiency is 80 percent, find
the time required. Ans: 32.9 s
3. A man uses a horizontal force of 200 N to push a crate up a ramp 8
m long that is 200 above the horizontal. (a) How much work does
the man perform? (b) If the man takes 12 s to push the crate up the
ramp, what is his power output in watts and in horsepower? Ans:(a)
1504 J (b) 125 W, 0.17 hp
WORK AND ENERGY THEOREM

 If a particle of mass m moves from x1 to x2, the magnitude of its


displacement is s = x2 – x1. Its final velocity v2 if it starts at v1 is
 v22 = v12 + 2a(x2 – x1) = v12 + 2as
a = v22 – v12/2s
 From the formula F = ma,
1 1 1
F = m(v22 – v12)/2s =( mv22 - mv12)/s where K = mv2
2 2 2

K = kinetic energy = energy possessed by a body in motion


1 1
 Fs = mv22 - 2 mv12
2

W = K2 – K1
W = ΔK →This is the work-energy theorem formula.
WORK-ENERGY THEOREM:EXAMPLES

 1. A constant force acts on a body of mass 4 kg and changes its speed from 4 m/s
to 8 m/s in a distance of 20 meters. How much force acts on the body?
 W = K 2 – K1
1 1 1 1
 Fs = 2
mv22 - 2 mv12 = 2 (4 kg)(8 m/s)2 - 2
(4 kg)(4 m/s)2
 F(20 m) = 128 J – 32 J = 96 J
96 𝐽
 F= = 4.8 N
20 𝑚

 2. A 100-kg moving cheetah has a kinetic energy of 20,000 J. What is the speed of
the cheetah?
1 1 40000 𝐽
 K = 2 mv2 → 20,000 J = 2 (100 kg)v2 → v2 = 2(20,000 J)/100 kg →v= 100 𝑘𝑔

 V = 20 m/s
WORK-ENERGY THEOREM: EXERCISES

1. A limousine is moving at a speed of 108 km/h. If the mass of the limo, including
its passengers, is 2000 kg, what is its kinetic energy? Ans: 9 X 105 J
2. Two cars are moving. The first car has twice the mass of the second car but only
half as much kinetic energy. When both cars increase their speed by 5 m/s, they
can have the same kinetic energy. Calculate the original speeds of the two cars.
5 2
Ans.: v1 = m/s, v2 = 5 2 m/s
2
3. A force of 1000 N is applied to a 50-kg body to change its speed from 10 m/s to 20
m/s. To what distance does the body move? Ans: 7.5 m
4. A bullet moving at a speed of 153 m/s passes through a plank of wood. Its speed is
130 m/s. Another bullet, of the same mass and size but moving at 92 m/s, passes
through an identical plank. What will the second bullet’s speed be after passing
through the plank? Assume that the resistance offered by the plank is independent
of the speed of the bullet.(Hint: Set up two equations to solve for the unknown
speed.) Ans: 44.2 m/s
POTENTIAL ENERGY

Potential energy(U) – the energy possessed by a body by virtue of its


position or its configuration.
Gravitational Potential Energy(Ug) – the potential energy of a body due
to its position. m
Ug = mgy
where m = mass of the body y
g = 9.8 m/s2
y = the vertical distance
At m = 10 kg and y = 50 m,
Ug = mgy = 10 kg (9.8 m/s2)(50 m) = 4900 J
ELASTIC POTENTIAL ENERGY

Elastic Potential Energy(Ue) - the potential energy of a body due to its


configuration.
1
Ue = 2kx2
where k = force constant (in N/m)
x = the elongation or compression(in meter)
1
At k = 5000 N/m and x = - 0.2 m, Ue = (5000 N/m)(-0.2 m)2 = 100 J
2

The value of x could either be positive or negative. A positive value denotes


elongation while a negative value denotes compression.
Conservation of Mechanical Energy

All forces are either conservative or nonconservative.


Conservative force – a force for which the work-energy relationship is
completely reversible. It is a force which allows stored energy, such as
potential energy, to be converted to kinetic energy and with the latter to
be retrieved again as potential energy. (Ex.: gravity, spring force,
electrical force, and magnetic force)
Four properties of the work done by a conservative force:
1. It can be expressed as the difference between initial and final values of
a potential-energy function. [W = -ΔU = -(U2 – U1) = U1 – U2]
2. It is reversible.
3. It is independent of the path of the body and depends on only the
starting and ending points.
4. When the starting and ending points are the same, the total work is
zero.
CONSERVATION OF ENERGY: A CONTINUATION

Nonconservative force – a force that is not conservative. Examples of


this force are friction and fluid resistance(viscosity). Some
nonconservative forces cause mechanical energy to be lost or
dissipated; a force of this kind is called a dissipative force.

WT = W1 + W2 = 0
W1
W WT = total work
W2 W = W1= - W2


MECHANICAL ENERGY: ITS PRICIPLE AND
CONSERVATION
Mechanical energy(E) – the sum of the kinetic energy(K) and potential
energy(U) of a body.
E=K+U
Principle of Conservation of Mechanical Energy: If the only force acting
on the body is conservative, the mechanical energy of the body
is conserved or it remains constant.
ΔE = E2 – E1 = 0 or ΔE = 0 or E2 = E1
If ΔE = 0, it follows that ΔE = ΔK + ΔU = 0 and ΔK = - ΔU. From the
latter, we can show that K2 - K1 = - (U2 – U1) or K2 - K1 = U1 – U2;
1 1
therefore, 2 mv2 - 2 mv12 = mgy1 – mgy2 which can be rearranged into
2
1 1
mv2 2+ mgy2 = mv12 + mgy1. The last is the equation for the principle of
2 2
MECHANICAL ENERGY: EXAMPLES

1. A stone is dropped from a height of 50 meters. What will be the


magnitude of its final velocity? Given: y1 = 50 m, y2 = 0, v1 = 0, and v2
=?
1 1
mv22 + mgy2 = mv12 + mgy1
2 2
1 1
2
mv 2
2 + m(9.8 m/s2)(0) = 2
m(0) 2 + m(9.8 m/s2)(50 m)
1 2
v + 0 = 0 + 490 m2/s2
2 2
𝑚2
v2 = 2(490 𝑠2 ) = 980 𝑚2/𝑠2

v2 = 31.3 m/s
MECHANICAL ENERGY: A CONTINUATION(PROB 1)

The previous problem can also be solved using a kinematic


equation. We can use v2 = vo2 – 2g(y – yo). Hence,
v2 = (0)2 – 2(9.8 m/s2)(0 – 50 m)
v2 = 980 m2/s2
v= 980 𝑚2/𝑠2
v = 31.3 m/s
Based on what is shown above, we can say that the equation for
conservation of mechanical energy can be used as an alternative for
kinematic problems as long as only conservative forces act on the body
or system of bodies.
MECHANICAL ENERGY: EXAMPLES

2. A 2-kg block moves toward a spring(k = 5000 N/m) at 5 m/s. Compute (a)
the compression of the spring if the surface is frictionless, (b) the final elastic
potential energy of the system(spring and the block).
(a) Using K2 + U2 = K1 + U1,
1 1 1 1
mv22 + 2 kx22 = 2 mv12 + kx12
2 2
1 1 1 1
(2 kg)(0)2 + (5000 N/m)x22 = 2 (2 kg)(5 m/s)2 + 2 (5000 N/m)(0)2
2 2

𝑚2
2 25 1 1
x2 = ± 𝑠2
b) U2 = 2 kx22 = (5000 N/m)(-0.1m)2
5000 𝑁/𝑚 2

x2 = - 0.1 m U2 = 25 J
MECHANICAL ENERGY: EXAMPLES

k = 5000 N/m
5 m/s
2 kg
3. A pendulum bob(0.4 kg) is swung to the left and it reaches a height of 0.2
m from its equilibrium position. If the bob is released and it swings back to its
equilibrium position and up to a certain height, (a) what is its speed upon
reaching its equilibrium position, (b) what is its final kinetic energy at that
position, and (c) how far above its equilibrium position does it swing up?
1 1
(a) K2 + U2 = K1 + U1 → mv22 + mgy2 = 2 mv12 + mgy1
2
1 𝑚2
2
v22 + 0 = 0 + (9.8 m/s2)(0.2 m) → v2 = 2 1.96 𝑠2 = 1.98 m/s
MECHANICAL ENERGY: A CONTINUATION(PROB3)

1 1
(b) K2 = mv22 = 2(0.4 kg)(1.98 m/s)2 = 0.784 J
2
1 1
(c) 2
mv 2
2 + mgy2 = 2
mv 1
2 + mgy
1
1 2 + m(9.8 m/s2)y = 1m(1.98
m(0) 2 m/s)2 + m(9.8 m/s2)(0)
2 2
𝑚 2
1.98 𝑠
y2 = 𝑚 = 0.2 m
2(9.8 )
𝑠2

0.2 m

equilibrium position
MECHANICAL ENERGY: EXAMPLES

4. A spring(k = 1000 N/m) is compressed (- 0.1 m) by placing a 2-kg


block on an incline(θ = 300) as shown in the figure. How far up the
incline does the block move?
K2 + U2g +U2e = K1 + U1g + U1e
1 1 1 1
m(0) 2 + mgy2 + k(0) 2 = m(0) 2 + mg(0) + kx1
2
2 2 2 2
1
0 + (2 kg)(9.8 m/s2)y2 +0=0+0+ (1000 N/m)(-0.1 m)2
2
19.6 N(lsin 300) = 5 Nm
5 𝑁𝑚
l= 19.6 𝑁(sin 300)

l = 0.51 m
MECHANICAL ENERGY: A CONTINUATION(PROB 4)

This is a figure for example 4.

y2 = lsin θ

θ
MOMENTUM: IMPULSE AND COLLISIONS
Momentum(p) – the product of the mass(m) and the velocity(v) of a
body.
p = mv (unit: kg m/s)
A 10-kg body with a magnitude of its velocity at 20 m/s has a
momentum of magnitude p = (10 kg)(20 m/s) = 200 kg m/s.
From Newton’s Law,
Δ𝑝
F = ma F= (the rate of change of p)
Δ𝑡
Δ𝑣 𝑣2 − 𝑣1
F= m = m( ) FΔt = Δp = the product of F and Δt
Δ𝑡 Δ𝑡
𝑚𝑣2 − 𝑚𝑣1 𝑝2 − 𝑝1
F= = J = Δp where J = impulse
Δ𝑡 Δ𝑡
MOMENTUM: EXAMPLES
1. A 5-ton car(including its passengers) drives through a freeway with a
momentum of 120,000 kg m/s. What is the magnitude of the velocity
of the car? (1 ton = 1000 kg)
𝑝 120000 𝑘𝑔 𝑚/𝑠
p = mv →v= = = 24 m/s = 86.4 km/h
𝑚 5000 𝑘𝑔
2. A body of mass m, moving at a velocity of magnitude v1, increases its
velocity to twice(2v1) its initial velocity. (a) What is the ratio of its initial
momentum to the final? (b) How about its kinetic energies?
1
𝑝1 𝑚𝑣1 𝑣1 1 𝐾1 𝑚𝑣12 𝑣1 2 1
2
(a) = = = (b) = 1 = =
𝑝2 𝑚𝑣2 2𝑣1 2 𝐾2 𝑚𝑣2
2 2𝑣1 2 4
2
Hence, p1 = p2/2 or p2 = 2p1 Hence, K1 = K2/4 or K2 = 4k1
IMPULSE: AN EXAMPLE
A ball of mass 200 grams is moving horizontally to the left at 10 m/s
when it hits a vertical wall which causes the ball to rebound at 8 m/s.
Assuming that the ball was in contact with the wall for 0.05 second, (a) What
is the impulse of the force exerted by the wall, and (b) what is the average
force exerted on the ball by the wall? (Hint: Take the rightward direction as
positive.)
𝑠 𝑚
(a) J = Δp = mv2 – mv1 = m(v2 – v1) kg m/s( ) = (kg 2) s = Ns
𝑠 𝑠
= 0.20 kg(8 m/s – (-10 m/s))
J = 3.6 N.s
𝐽 3.6 𝑁.𝑠
(b) J = FΔt → F= = = 72 N
Δ𝑡 0.05 𝑠
THE PRINCIPLE OF CONSERVATION OF
MOMENTUM
For any system, the forces that the particles of the system exert on
each other are called internal forces. Forces exerted on any part of the
system by some object outside it are called external forces. For a system
where there are no external forces acting, then it is described to be an
isolated system.
Principle of Conservation of Linear Momentum: If the vector sum of
the external forces on a system is zero, the total momentum of the system is
constant or it is conserved.
momentum before = momentum after
This principle is used to explain: (1) the recoil of a gun, a tank, and a cannon,
(2) the upward thrust of a rocket, (3) the backward movement of skaters
when they push against each other, (4) the forward erratic movement of the
balloon when its nozzle is opened, etc.
CONSERVATION OF MOMENTUM: EXAMPLES
A hunter holds a rifle of mass mR = 3 kg loosely, so it can recoil
freely. He fires a bullet of mass mB = 5 g horizontally with a velocity
relative to the ground of vB = 300 m/s. (a) What is the recoil velocity of
vR of the rifle? (b) What is the final momentum and kinetic energy of
the bullet and rifle?
(a) momentum before firing = momentum after firing
0 = mRvR + mBvB
𝑚
𝑚𝐵𝑣𝐵 (0.005 𝑘𝑔)(300 𝑠 )
- vR = =
𝑚𝑅 3 𝑘𝑔
vR = - 0.50 m/s
1 1
(b) p = mRvR + mBvB = K= mvR2 + mvB2 =
2 2
COLLISIONS AND MOMENTUM
CONSERVATION
Collisions in physics encompass any strong interaction between bodies
that lasts a relatively short time. If the forces between colliding bodies are
much larger than any external forces, as in the case in most collisions, we can
ignore the external forces and treat the bodies as an isolated system.
Elastic and Inelastic Collisions
If the forces between the colliding bodies are also conservative, so no
mechanical energy is lost or gained in the collisions, the total kinetic energy
of the system is the same after the collision as before. Such a collision is
called an elastic collision.
A collision in which the total kinetic energy after the collision is less
than before the collision is called an inelastic collision. An inelastic collision in
which the colliding bodies stick together and move as one after the collision
is called a completely(or perfectly) inelastic collision.
COLLISIONS: SAMPLE PROBLEMS
1. A 3.2-kg object with a speed of 15 m/s has a completely inelastic
collision with a 4.8-kg object initially at rest. Find the final speed of
the combination.
2. A boy throws a 3.3-kg ball to a 48-kg girl on roller skates who is
initially stationary. After catching the ball, she starts moving at 0.32
m/s. How fast was the beach ball going when she caught it?
3. A 3.2-kg object with a speed of 15 m/s has a head-on elastic
collision with a 4.8-kg object initially at rest. Find the speeds of the
objects after collision.
COLLISIONS: SOLUTIONS
1. momentum before collision = momentum after collision
mAvA + mBvB = mAV + mBV = V(mA + mB)
(3.2 kg)(15 m/s) + (4.8 kg)(0) = V(3.2 kg + 4.8 kg) = V(8.0 kg)
48 kg m/s = V(8.0 kg)
6 m/s = V
2. 3.3 kg(vA) + 0 = 0.32 m/s(3.3 kg + 48 kg)
16.416 𝑘𝑔 𝑚/𝑠
vA =
3.3 𝑘𝑔
vA = 4.97 m/s
3. momentum before = momentum after kinetic energy before = kinetic energy after
1 1 1 1
mAv1A + mBv1B = mAv2A + mBv2B mAv1A + mBv1B = mAv2A + mBv2B2
2 2 2
2 2 2 2
3.2 kg(15 m/s) + 4.8 kg(0) = 3.2 kg(v2A) + 4.8 kg(v2B) (3.2 kg)(15 m/s)2 + 0 = 3.2 kgv2A2 + 4.8kgv2B2
48 kg m/s = 3.2kg(v2A) + 4.8 kg(v2B) 720 kg m2/s2 = 3.2 kgv2A2 + 4.8 kg v2B2
1 2
COLLISIONS: SOLUTIONS 2
𝑚
48 𝑘𝑔 − 4.8 𝑘𝑔 𝑣2𝐵
𝑠
From eq. 1, v2A = = 15 m/s – 1.5 v2B → Substitute this into eq. 2.
3.2 𝑘𝑔
720 kg m2/s2 = 3.2 kg (15 m/s – 1.5 v2B)2 + 4.8 kg v2B2
= 3.2 kg (225 m2/s2 – 45 m/s v2B + 2.25 v2B2) + 4.8 kg v2B2
720 kg m2/s2 = 720 kg m2/s2 – 144 kg m/s v2B + 12 kg v2B2
- 12 kg v2B2 = - 144 kg m/s v2B
v2B = 12 m/s Checking:
1 1
Substitute v2B = 12 m/s into eq. 1, (3.2 kg)(15 m/s)2 +0= (3.2 kg)(-3 m/s)2 +
2 2
1
48 kg m/s = 3.2 kgv2A + 4.8 kg(12 m/s) (4.8 kg)(12 m/s)2
2
48 kg m/s = 3.2 kgv2A + 57.6 kg m/s 360 kg m2/s2 = 14.4 kg m2/s2 + 345.6 kg m2/s2
v2A = - 3 m/s 360 J = 360 J
MOMENTUM: EXERCISES
1. A 1.5-kg basketball bounces off a backboard. Initially it was going 12 m/s
horizontally directly toward the backboard. It bounces straight back with its
speed unchanged. What is its change in momentum? (Ans.:Δp = 36 kg m/s)
2. A 50-kg girl dives off a 1000-kg boat. She leaves the boat with horizontal speed
5.2 m/s. Assume the boat is originally at rest and free to move in the water.
With what speed does the boat start to move off?(Hint: This is similar to the
rifle-bullet problem.)(Ans.:vB = - 3.85 m/s)
3. A machine gun fires 40-g bullets at speed of 500 m/s. The gunner, holding the
machine gun in his hands, can exert a maximum force of 200 N against the gun.
Determine the maximum number of bullets he can fire per minute. (Hint: Solve
Δ𝑝
for the impulse = Δp, then F1 = = force for one(1) bullet per Δt = 1 second.
Δ𝑡
No. of bullets/min = (200 N /F1)X 60 (s/min)= 600/min)
MOMENTUM: EXERCISES 2
4. A 40-kg skater moving at 4 m/s overtakes a 60-kg skater moving at 2 m/s in the
same direction and collides with him. (a) If the two skaters remain in contact, what
is their final velocity? (b) How much kinetic energy is lost?(Hint: In (b) solve for the
initial kinetic energy minus the final kinetic energy). Ans.: (a) v = 2.8 m/s, (b) K = Ki
– Kf = 48 J.
5. A 4-kg object moving to the right at 5 m/s collides head-on with a 2- kg object
moving to the left at 7 m/s. (a) If the collision is elastic, find the respective
velocities after collision and (b) show that their kinetic energies before and after
collision are equal. Ans.: (a) v2A = - 3 m/s and v2B = 9 m/s
ROTATIONAL MOTION
Angles can be measured either in radian(rad) or in degrees, where 3600 is one
revolution(rev). If a circle is drawn whose center is at the vertex of a particular angle, the
angle θ in radians is equal to the ratio between the arclength s cut by the angle and the
radius r of the circle.
𝑠 𝑎𝑟𝑐𝑙𝑒𝑛𝑔𝑡ℎ
θ= =
𝑟 𝑟𝑎𝑑𝑖𝑢𝑠
3600 = 2π rad 1 rev = 3600 = 2π rad
1800 = π rad ? rad
5 rev = _____
1800 2π𝑟𝑎𝑑
1 rad = 5 rev( ) = 10π rad
π 1 𝑟𝑒𝑣
1 rad = 57.30 5 rev = 10π rad
The unit radian has no θ s
dimension because it is a ratio
of two lengths.
ROTATIONAL MOTION
What is the equivalent of 300 in radians? 750 in radians? 2700 in radians?
0 π 𝑟𝑎𝑑 π 0 π 𝑟𝑎𝑑 5 0 π 𝑟𝑎𝑑 3
30 ( )
0 = rad 75 ( )
0 = π rad, 270 ( )
0 = π rad
180 6 180 12 180 2

2 4
What is the equivalent of π rad in degrees? π rad in degrees?
3 5
2 1800 4 1800
π rad( ) = 1200 π rad( ) = 1440
3 π 𝑟𝑎𝑑 5 π 𝑟𝑎𝑑
ROTATIONAL MOTION
angular velocity(ω) – the rate of change of the angular displacement(θ) of a
rotating body.
average angular velocity(ωav) – the change in angular displacement(Δω) over
the time interval(Δt).
θ2 − θ1 Δθ
ωav = = (unit: rad/s or rev/s)
𝑡2 −𝑡1 Δ𝑡
instantaneous angular velocity(ω) – the angular velocity at a certain instant.
Δθ 𝑑θ
ω= lim =
Δ𝑡→0 Δ𝑡 𝑑𝑡
angular acceleration(α) – the rate of change of the angular velocity(ω) of the
rotating body.
ROTATIONAL MOTION
average angular acceleration(αav) – the change in angular velocity(Δω)
over the change in time(Δt).
ω2 − ω1 Δω
αav = = (unit: rad/s2 or rev/s2)
𝑡2 − 𝑡1 Δ𝑡
instantaneous angular acceleration(α)
Δω 𝑑ω
α= lim =
Δ𝑡→0 Δ𝑡 𝑑𝑡
where: θ is read as “theta”
ω is read as “omega”
α is read as “alpha”
ROTATIONAL MOTION
CONSTANT ANGULAR ACCELERATION
Linear Motion Rotational Motion
1. v = vo + at ω = ωo + αt
1 2 1 2
2. x = xo + vot + at θ = θo + ωo t + αt
2 2
3. v2 = vo2 + 2a(x – xo) ω2 = ωo2 + 2α(θ - θo)
𝑣𝑜 + 𝑣 ω𝑜 + ω
4. x = xo + ( )t θ = θo + ( )t
2 2
𝑠
From θ = , we get s = rθ. By differentiation, ds = r dθ. Then divide by dt. We
𝑟
𝑑𝑠 𝑑θ 𝑑𝑣 𝑑ω
have = r which is equivalent to v = rω. Differentiating again, = r
𝑑𝑡 𝑑𝑡 𝑑𝑡 𝑑𝑡
which equivalent to a = rα. (v = linear speed, a = linear acceleration)
ROTATIONAL MOTION: SAMPLE PROBLEMS
1. A wheel starts from rest at a constant angular acceleration and turns
through 500 radians in 10 seconds. (a) What is the angular acceleration of
the wheel? (b) What is the final angular speed?
given : ωo = 0, θo = 0, θ = 500 rad, and t = 10 s
1 ω𝑜 + ω
(a) θ= θo + ωot + αt2 (b) θ = θo + ( )t
2 2
1 0+ω
500 rad = 0 + 0 + α(10 s)2 500 rad = 0 + ( )(10 s)
2 2
1 ω
500 rad = α(100 s2) 500 rad = (10 s)
2 2
10 rad/s2 = α 100 rad/s = ω
ROTATIONAL MOTION: EXAMPLE 2
2. An engine idling at 10 rev/s is accelerated at 2.5 rev/s2 to 20 rev/s. (a) How many
revolutions does it make during this acceleration? (b) How long does it take to
reach 20 rev/s? Given: ωo = 10 rev/s, α = 2.5 rev/s2, and ω = 20 rev/s
(a) ω2 =ωo2 + 2α(θ – θo) (b) ω = ωo + αt
(20 rev/s)2 = (10 rev/s)2 + 2(2.5 rev/s2)(θ - 0) 20 rev/s = 10 rev/s + (2.5 rev/s2)t
400 rev2/s2 = 100 rev2/s2 + (5.0 rev/s2)θ -(2.5 rev/s2)t = 10 rev/s – 20 rev/s
− 10 𝑟𝑒𝑣/𝑠
- (5.0 rev/s2)θ = 100 rev2/s2 – 400 rev2/s2 t=
− 2.5 𝑟𝑒𝑣/𝑠2
− 300 𝑟𝑒𝑣2/𝑠2
θ= t=4s
− 5.0 𝑟𝑒𝑣/𝑠2
θ = 60 rev
ROTATIONAL MOTION: EXAMPLE 3
3. The blades of a rotary lawnmower is 30 cm long and rotate at 315 rad/s. Find (a)
the linear speed of the blade tips, (b) the angular speed in rev/s, and (c) the linear
distance it travels in one minute. (Given r = 30 cm = 0.3 m, and ω = 315 rad/s.)
θ
a) v = rω = 0.3 m(315 rad/s) c) s = rθ where ω = → θ= ωt
𝑡
v = 94.5 m/s θ= (315 rad/s)(60 s)= 18900 rad
1 𝑟𝑒𝑣
b) ω = 315 rad/s( ) Hence, s = 0.3 m(18900 rad) = 5670 m
2π𝑟𝑎𝑑
ω = 50 rev/s s = 5670 m

Note: In this problem the angular speed ω is constant since it is not specified that
the blades are rotating at constant angular acceleration. Also, the unit radian can
just be omitted because it has no dimension and it is a ratio of two lengths.
What does it take to start a stationary body rotating(or having an angular
velocity) or to bring a spinning body to a halt? A net force is required, but it must
be applied in a way that gives a twisting or turning action. That twisting or
turning effect of the force is called torque(τ) or moment of force in engineering
parlance. Torque is defined by the formula
τ=rxF
or in its magnitude form,
τ = rFsin φ
where F is the net force and rsin φ = l is the moment arm(or lever arm). At φ=
900, r = l , therefore, τ = Fl. The unit of torque is Nm but it is not the same as
Joule. (The Greek letter τ(read as tau) represents torque in physics.)
TORQUE: A CONTINUATION

Sign convention: A counterclockwise


torque is positive while a clockwise F
torque is negative. Fsin φ

line of action of the force

axis of rotation

At F = 200 N and r = 5m and φ = 53.10 , the torque is τ = 5 m(200 N) sin 53.10 = 800 Nm.
The torque is positive because it causes the rod to rotate counterclockwise.
CONDITIONS FOR EQUILIBRIUM
For a body or system to be in mechanical equilibrium it must meet two
conditions. First, there must be no net force acting on the body or ΣF = 0 and
second, there must be no net torque on the body or Στ = 0. At this
mechanical equilibrium, the body is either at rest or moving at constant
velocity. For example, person A(mA = 80 kg) sits at one end of a 4-m seesaw
while person B(mB = 60 kg) sits at the other end. Where should the third
person C(mC = 50 kg) sit so that the seesaw is in equilibrium?

WA WC WB
2m x
2m
pivot(lever)
TORQUE: EXAMPLES
For the seesaw to be in equilibrium,
Στ = 0
- WB(2 m) - WC(x) + WA(2 m) = 0
- mBg(2 m) - mCg(x) = -mAg(2 m) → g cancels out of the equation
(60 kg)(2 m) + (50 kg)(x) = 80 kg(2 m)
50 kg)(x) = 160 kg m – 120 kg m
40 𝑘𝑔 𝑚
x=
50 𝑘𝑔
x = 0.8 m
Therefore, person C must sit 0.8 m from the pivot of the seesaw.
TORQUE: EXAMPLES
2. The uniform horizontal 100-kg beam shown in the figure is 6 m long and is
pivoted at a point in the wall. The other end of the beam is supported by a
rope which makes 300 with the horizontal. Find(a) the tension in the rope
and (b) the horizontal and vertical components of the force exerted by the
wall at the pivot.
Fv T Tsin 300
L
300
Fh Tcos 300
100 kg Wb WB
axis
300 kg
TORQUE: EXAMPLE 2
Since the beam is in equilibrium, Στ = 0; therefore,
1
Στ = T Lsin 300 - WBL – Wb( )L = 0
2
1
T L sin 300 = WBL + WbL
2
1
T sin 300 = mBg + mbg
2
𝑚 1 𝑚
300 𝑘𝑔 9.8 𝑠2 + 2(100 𝑘𝑔)(9.8 𝑠2 )
T=
sin 300
T = 6,860 N ΣFy = 0
ΣFx = 0 Fv + T sin 300 – WB –Wb = 0
Fh – T cos 300 = 0 Fv + 3430 N - 2940 N – 980 N =0
Fh = T cos 300 = 6860 N(0.866) = 5941 N Fv = 490 N
TORQUE: EXAMPLES

𝐹𝑣 490 𝑁
F2 2
= Fh + Fv2 tan θ= =
𝐹ℎ 5941 𝑁
490
F = 5941 𝑁 2 + 490 𝑁 2 θ= tan-1( )= 4.70
5941
F = 5961 N → The force exerted by the wall on the
beam.
3. A 5-m uniform ladder weighing 120 N is placed against a smooth wall in such a way that
it makes an angle of 600 with the horizontal. A man weighing 480 N climbs up the ladder
and stays 1.3 m from its upper end. Find (a) the force exerted by the wall against the
ladder and (b) the horizontal and the vertical components of the force exerted by the floor
on the lower end of the ladder.

600
TORQUE: EXAMPLE 3
Fw where:
Fw = the force exerted by the wall on the ladder
Wm = the weight of the man
5 m sin 600 WL = the weight of the ladder
Wm
Point O is the axis of rotation.

Fv WL Στ = 0
600 Fw(5 m sin 600) – WL(2.5 m cos 600) –
O Fh 2.5 m cos 600 Wm(3.7 m cos 600) = 0
Fw(4.33 m) – 120 N(1.25 m) – 480 N(1.85 m) = 0
TORQUES: EXAMPLE 3(CONT.)
Fw(4.33 m) – 150 Nm – 888 Nm = 0
Fw(4.33 m) = 990 Nm
1038 𝑁𝑚
Fw =
4.33 𝑚
Fw = 240 N
ΣFx = 0 ΣFy = 0
F h – Fw = 0 Fv – WL – Wm = 0
F h = Fw Fv – 120 N – 480 N =0
Fh = 240 N Fv – 600 N =0
Fv = 600 N
CONDITIONS FOR EQUILBRIUM
1. First Condition: The vector sum of all the external forces acting on an object or a system of
objects is zero.
ΣF = 0 ΣFx = 0 This condition must be met to ensure that the
ΣFy = 0 object does not accelerate.
ΣFz = 0

2. Second Condition: The vector sum of all the toques on the object or system of objects is zero.
Στ = 0 This condition must be met to ensure that the object
does not rotate.
Chapter 10 Lecture Notes
Physics 2414 - Strauss

Formulas:
ρ = m/V
Pfluid = ρgh
Ptotal = ρgh +P0 = PG +PA
F 2 = F 1 (A 2 /A 1)
B = wfluid
Q = ∆V/∆t =Av (Flow rate)
ρ 1A 1v 1 = ρ 2A 2v 2
P1 + (1/2)ρv12 + ρgy1 = P2 + (1/2)ρv22 + ρgy2
F =ηAv/l

Main Ideas:
1. Definition
2. Pressure
• Definitions and Units
• Properties of Pressure in a Fluid
• Pascal’s Principle
• Measurements of Pressure
3. Buoyancy
4. Motion of Fluids
• Bernoulli’s Equation
• Viscosity
1. Definition of Fluids
In this chapter we study fluids. A fluid is a defined as substance that can flow.,
that doesn’t maintain a fixed shape. Gases and liquids are usually considered
fluids. Any object, whether a solid, a gas, a liquid, or a plasma (a collection of
ionized particles), has a density. The density is defined as the mass per unit
volume and is given the Greek symbol rho (ρ). So the density is defined as

ρ = m/V. (kg/m3)

A substance which is more dense will have more mass for a given volume. If I
hold a cubic centimeter of lead, it will be heavier than a cubic centimeter of wood
because lead has a higher density than wood. Table 10-1 gives the density of
some materials.

Another way of specifying the density of an object is to compare its density to


the density of water at 4.0°C, (which is 1.00 × 103 kg/m3). The specific
gravity is defined as the ratio of the density of a substance to the density of

1
water at 4.0°C. Lead, with a specific gravity of 11.3, has a density of 11.3 × 103
kg/m3.
2. PRESSURE
2.1 Definition and Units
Pressure is defined as the force per unit area. P = F/A. The force acts
perpendicular to the surface area A. If a person stands on a wooden floor with
spiked metal cleats, the floor will be damaged because the person’s weight is not
spread out over a very large area. However, if the person wears normal shoes,
the weight is spread out over a larger area, so the pressure is smaller, and the
floor will not be damaged. Snow shoes spread the weight out over an even larger
area, and so the pressure on the snow is less, and a person does not sink into the
snow. A person can lay on a bed of nails because there are so many nails that the
person’s weight is still spread out over a lot of area, and the pressure is not very
high.

The SI units of pressure is given by N/m2 which is given the name a Pascal (Pa),
named after the French philosopher, theologian, and scientist Blaise Pascal.

1 N/m2 = 1 Pa.

In the U.S. we also use lb/in2 or psi (pounds per square inch).

Because the atmosphere is always pushing down on us with a certain pressure,


another common unit of measurement, often used when discussing fluids, is called
an atmosphere. This is defined as the standard pressure that the atmosphere
presses on us at sea level and is equal to 1.013 × 105 Pa. This is given the
pressure unit of atmospheres.

Finally, we will see that other fluids can push on us just like the air pushes on us.
A commonly used measure of pressure comes from the fact that if I were
immersed in a column of mercury 760 mm high, that would exert the same
pressure on me as the atmosphere. We give this unit of mm/Hg (millimeters of
mercury) a special name called a torr. So

1 atmosphere = 760 torr (mm/Hg) = 1.013 × 105 Pa

Problem: A water bed is 2.00 m square and 30.0 cm deep. (a) What is its
weight? (b) What pressure does the bed exert on the floor, assuming the entire
lower surface contacts the floor? (Volume = length × width × height, Area =
length × width).

2
2.2 Properties of Pressure in a Fluid
There are three important observations about pressure in a fluid.

1) At any point that a fluid is in contact with a surface, the pressure is directed
perpendicular to the surface.

2) At any point inside a fluid, the pressure is directed in all directions with the
same magnitude. (See figure 10-1). For the block in this figure, the pressure is
not exactly the same because the different faces are at different depths. But if the
block is infinitesimally small, then the pressure in all directions is exactly the
same.

3) The pressure at any point in a fluid depends only on the depth of the point.
Suppose there is a volume of fluid with a uniform density which has a depth h
and area A.

What forces act on this volume of fluid? F 0 = P 0A


There is the force from the atmosphere
above the liquid F0 = P0A. There is the
weight of the liquid Mg. There is also the h
force of the liquid pushing up on the
column of liquid at depth h. Mg PA

Since there is no acceleration of the liquid at the


depth h, the forces must balance each other.
ΣFy = 0
PA -Mg - P 0A = 0
PA -ρ Vg - P 0A = 0
PA - ρ hAg - P 0A = 0

P = ρgh +P0

So the pressure at any depth h in a fluid is equal to the pressure outside of the
fluid (P0) plus the fluid pressure (ρhg).

For many circumstances, the pressure outside of the fluid is the pressure of the
earth’s atmosphere at that point. For instance, the pressure in your tires is really
the pressure of the air in your tires plus the pressure of the atmosphere on the
tires. However, the pressure that is read on a pressure gauge is the pressure
which is greater than the atmospheric pressure. It is called gauge pressure. In
general, the equation above becomes

3
P = PG +PA

where PG is the gauge pressure and PA is the atmospheric pressure. We are often
most concerned with the gauge pressure or fluid pressure, and not the outside
pressure. For fluids, then, we often simply write the fluid pressure, or

Pfluid = ρgh

Problem: A rectangular shaped dam is 70 m high and 180 m wide and water is
filled to the top of the dam. (a) What is the pressure on the dam from the water
at the top? (b) What is the pressure on the dam from the water at the bottom?
(c) What is the average pressure on the dam from the water? (d) What is the
total force on the dam? (e) What if we include the pressure from the
atmosphere? (f) Suppose that the dam was holding up a thin column of water
instead of a huge lake. What would be the pressure on the dam then?

2.3 Pascal’s Principle


Note that the pressure is only dependent on the depth, and not on the “area” of the
column of fluid. This means that the pressure at any depth of a fluid is always
the same for all points at that depth. The pressure is not affected by the shape of
the vessel. The pressure under 5 meters of water in a thin tube is the same as the
pressure under 5 meters of water in a big swimming pool. This is why water
always seeks a constant level. The pressure at any uniform height in a single
liquid is the same throughout the entire liquid.

In addition, if the outside, or external pressure increases, the total pressure


increases by the same amount. Pascal realized that this led to an important
general principle. Pascal’s principle states that pressure applied to a
confined fluid increases the pressure throughout the fluid by the same
amount. This is how a hydraulic lift works. Fluid is enclosed in a pipe with a
small area at one end and a large area at the other. Pressure is applied at the end
with the small area. That same pressure is transferred to the end with the large
area.

P1 = F1 /A1 = P2 = F2 /A2
F 2 = F 1 (A 2 /A 1) F1 A1 F2 A2

If the area A 2 is much larger than the area A 1 , then a small force F1 can be
applied to create a large force F2 at the output end. This large force can be used
to jack up a car or lift heavy objects.

4
Problem: In the hydraulic press used in a trash compactor, the radii of the input
piston and the output plunger are 6.4 × 10-3 m and 5.1 × 10-2 m, respectively.
What force is applied to the trash when the input force is 330 N?
2.4 Measurements of Pressure
Pressure is measured using the two principles discussed above, that P =ρgh, and
Pascal’s Principle. If a fluid, like mercury or water is put in a container which is
open to the atmosphere at one end and closed at the other, with the closed end
having zero pressure (it is a vacuum), then the following situation occurs.

The pressure at any height is equal, so the P=0


pressure of the atmosphere, just equals the
pressure of the liquid or ρgh. h P = 1 atm

For different liquids with different


densities, the height of the column at sea level
will be different. For mercury it is 760 mm. For water, it is 10.3 m.

What happens when you suck on a straw? You take the air out of the straw and
the atmospheric pressure pushes the liquid up the straw. You do not suck the
liquid up. Instead you create a vacuum and the atmosphere pushes the liquid up.
Similarly, a water pump pumping at the top of a well can only pump water up
10.3 m, since it works the same way, and atmospheric pressure is 10.3 m of
water. To pump water up larger distances, you must use a series of pumps, or
place the pump at the bottom of the well, and then it pushes the water up, rather
than using atmospheric pressure to push it up.

3. BUOYANCY
Any object which is partially or totally submerged in a liquid has a buoyant force
acting on it which pushes the object up. That is why a rock appears to weigh less
when it is submerged in liquid, or why it is very difficult to push a beach ball
underwater. The famous Greek mathematician, Archimedes’ developed a
principle which describes this around 250 B.C. Archimedes’ principle can be
stated as any body completely or partially submerged in a fluid i s
buoyed up by a force equal to the weight of the fluid displaced by the
body.

B = wfluid

where B is the magnitude of the buoyant force and W fluid is the weight of the
displaced fluid. The reason for this is that the pressure of the fluid is dependent

5
on the depth of the fluid. So the pressure at the top of an object is less than the
pressure at the bottom of the object which creates a net force.

B = P 2A - P 1A
= (P 2 - P 1)A = ρ ghA
= ρ gV = mg =w h

Problem: (a) What is the buoyant force on a balloon filled with 1.0 m 3 helium at
sea level. (b) What is the gravitational force on the same balloon?

So the upward (buoyant) force is much greater than the downward force (weight)
and the helium balloon rises.

Problem: Every fluid exerts a buoyant force. Even the air exerts a buoyant
force on your body. Estimate how strong that force is.

Problem: Archimedes supposedly determined whether or not the king’s crown


was made of gold by using this law. Suppose a crown is weighed on a scale and it
weighs 9.8 N. The crown is then weighed in water and it weighs 9.00 N. Is the
crown pure gold?

Problem: A raft is made of wood having a density of 600 kg/m3. Its surface area
is 5.7 m2, and its volume is 0.60 m3. How much of it is below water level?

Problem: A hollow log is used as a canoe. It has a length of 3.00 m and a radius
of 0.350 m. The canoe weighs 1.00 × 103 N. What is the maximum weight it can
carry without sinking?

As the weight in the canoe is increased, it sinks lower in the water. The
maximum weight it can carry is when the water level is at the top of the canoe’s
side. The canoe has an upward buoyant force FB, and two downward forces, the
weight of the canoe wC, and the weight of the cargo, w.

4. MOTION OF FLUIDS
Up until this point, we have discussed fluids which are static. That is, they are
not in motion. We now turn our attention to fluids in motion, or hydrodynamics.
There are many categories of fluids in motion, categorized by whether the fluid
flow is steady, or not steady, compressible or incompressible, viscous or
nonviscous. In steady flow, the velocity of the fluid particles at any point is
constant as time goes by. Different parts of the fluid may be flowing at different
rates, but the fluid in one location is always flowing at the same rate. An

6
incompressible flow is the flow of a fluid which cannot be compressed. Most
liquids are nearly incompressible. A viscous fluid is one which does not flow
easily, like honey, while a nonviscous fluid is one which flows more easily, like
water. We will mostly be concerned with the steady flow of incompressible,
nonviscous fluids.

If the flow is steady, then the velocity of the fluid particles at any point is a
constant with time. The various layers of the fluid slide smoothly past each
other. This is called streamline or laminar flow. Above some certain
velocity, the flow is not smooth and becomes turbulent. Illustrations of
turbulent and laminar flow are shown in Figure 10-14.

We first consider the steady flow of a fluid through an enclosed pipe. We want
to determine how the speed of the fluid changes when the size of the pipe
changes.
At some point along the pipe, we
look at how much fluid flows past ∆l
us (∆m) in a short period of time (∆t). point 2
∆m/∆t = ρ ∆V/∆t = ρ A∆l/∆t = ρ Av point 1

This is the flow rate for any point along the pipe. Because no fluid flows in or
out of the sides, the mass flowing past any point during a short period of time
must be the same as the mass flowing past any other point, so

ρ 1A 1v 1 = ρ 2A 2v 2

where the subscript 1 and 2 refer to two different points along the pipe. This
equation is called the equation of continuity. If the fluid is incompressible,
then the density is the same at all points along the pipe and this equation becomes

A 1v 1 = A 2v 2 .

We see that if the cross sectional area is decreased, then the flow rate increases.
This is demonstrated when you hold your finger over part of the outlet of a
garden hose. Because you decrease the cross sectional area, the water velocity
increases.

The flow rate or flux is defined as the rate with which the fluid flows and is
given by the variable Q.

Q = ∆V/∆t =Av

7
Problem: A water hose with a radius of 1.00 cm is used to fill a 20.0 liter
bucket. If it takes 1.00 min to fill the bucket, what is the speed, v, at which the
water leaves the hose? (1.00 liters = 103 cm3)

Problem: The approximate inside diameter of the aorta is 1.0 cm (radius = 5.0 ×
10-3 m) and that of a capillary is 10 µm = 10 × 10-6 m. The approximate blood
flow speed is .3 m/s in the aorta and 0.5 mm/s in the capillaries. Approximately
how many capillaries get blood from the aorta?

4.1 Bernoulli’s Equation


In the 18th century, the Swiss physicists Daniel Bernoulli derived a relationship
between the velocity of a fluid and the pressure it exerts. Qualitatively,
Bernoulli’s principle states that swiftly moving fluids exert less pressure
than slowly moving fluids.

Bernoulli’s principle is extremely important in our everyday life. It is the


primary principle which leads to lift on an airplane wing and allows the plane to
fly. It is the primary reason a sailboat can sail into the wind. It is the primary
reason a baseball can curve. It is an important reason that smoke is drawn up a
chimney.

Airplane wing

Curve Ball Faster air

Slower Air

Bernoulli’s equation is really a consequence of a fundamental principle of


physics: the conservation of energy. It can be derived using energy principles.

Consider a fluid moving through a pipe. Point 2


The pipe’s cross sectional area changes,
and the pipe changes elevation. At one P2
point the pipe has a cross sectional area
of A1, a height of y1, a pressure of P1, ∆x 1 ∆x2
a velocity of v1 and moves a distance of
∆x1 in a time of ∆t. At another point P1
along the pipe these quantities are given Point 1
by A2, y2, P 2,v 2, and ∆x2.

8
We are going to push a certain amount of fluid up the pipe from point 1 to point
2. P1 is opposite in direction from P2 because the rest of the fluid pushes to the
left of fluid at point 2 and to the right of the fluid at point 1.

Recall, the fundamental equation that

W NC = ∆K + ∆U

We want to look at each of these terms individually to derive Bernoulli’s


equation.

∆K = (1/2)mv 22 - (1/2)mv 12 = (1/2)ρ V 2v22 - (1/2)ρ V 1v12

∆U = mgy 2 - mgy 1 = ρ V 2gy 2 - ρ V 1gy 1

W 1 = F∆x1 = P 1A 1∆x1
W 2 = F∆x2 = -P 2A 2∆x2

W NC = W 1 +W 2 = P 1A 1∆x1 - P 2A 2∆x2 = P 1V 1 - P 2V 2

W NC = ∆K + ∆U
P 1V 1 - P 2V 2 = (1/2)ρ V 2v22 - (1/2)ρ V 1v12 + ρ V 2gy 2 - ρ V 1gy 1

We know that since this is an incompressible fluid, V1 =V2

P1 - P2 = (1/2)ρv22 - (1/2)ρv12 + ρgy2 - ρgy1

P1 + (1/2)ρv12 + ρgy1 = P2 + (1/2)ρv22 + ρgy2

which is Bernoulli’s equation.

Remember, that Bernoulli’s principle applies when a fluid is moving. The


pressure is not the same if the velocities are different. Pascal’s principle applies
when the fluid is stationary. A pressure applied at one point is transferred to
every point of the fluid. Pascal’s principle can be used for something moving
slowly or something that moves a little then stops like a hydraulic jack.

As shown in the book, for a large storage tank of height h which has a small pipe
open at the bottom of it, the pressure at the top and at the small opening is
atmospheric pressure. This is actually an important point for solving problems.
The pressure of any part of a container open to the atmosphere is the same
pressure as the atmosphere for the entire section of container that is at the same

9
height as the opening and has the same cross sectional area as the opening. Also
in this case, the velocity at the top of the storage tank is basically zero because it
has such a huge cross sectional area, so Bernoulli’s equation gives the velocity at
the bottom of the tank to be:

P1 + (1/2)ρv12 + ρgy1 = P2 + (1/2)ρv22 + ρgy2


P1 = P2 = 1 atm, v1 = 0, y1 -y2 = h, so
v2 = √{2gh}

Problem: A water pipe is inclined 30° below the horizontal. The radius of the
pipe at the upper end is 2.00 cm. If the gauge pressure at a point at the upper end
is 0.100 atm, what is the gauge pressure at a point 3.00 m downstream where the
pipe has narrowed to 1.00 cm radius? The flow rate is 20.0π cm3/s.

Problem: A hypodermic syringe contains a fluid with the density of water. The
barrel of the syringe has a cross sectional area of 2.50 × 10-5 m2 and the cross
sectional area of the needle is 2.50 × 10-8 m 2. Before the plunger is pushed, the
pressure everywhere is at 1.00 atmospheres. A force, F, of 2.00 N is exerted on
the plunger. Assume that the gauge pressure in the needle remains at 1.00 atm.
If the syringe is horizontal, what is the speed of the liquid as it flows through the
needle and into the arm?

Don’t confuse this situation with one where there is a piston pushing on the fluid,
but the fluid doesn’t move. If you are considering the situation before the fluid
starts to move Pascal’s principle applies. After the fluid starts to move
Bernoulli’s principle applies.
4.2 Viscosity
Viscosity is the amount of internal friction in a fluid. As stated before, a fluid
like honey is very viscous, and a fluid like water has a low viscosity. The
viscosity of a fluid is quantitatively given by its coefficient of viscosityη (the
Greek letter, eta). It takes a constant force to move a fluid, because there is a
frictional viscous force opposing the motion. (Remember from Newton’s 2nd
law that if there was no frictional force, it would not take any external force to
keep a body moving at constant velocity). The force it takes to keep a liquid
moving in a tube is found to follow the equation

Section of fluid flowing


F =ηAv/l
immobile wall

where A is the area of the fluid in contact with the walls, v is the velocity of
fluid, and l is the perpendicular distance from the fluid to the immobile surface.

10
So a fluid has more drag near the surface of a stationary plate, and as l
approaches zero it takes an infinite force to move the fluid. In other words, the
fluid touching the wall does not actually move at all. Table 10-3 gives
coefficients of viscosity for some fluids.
5. SURFACE TENSION AND CAPILLARITY
We now deal with what happens at the surface of a fluid. The surface of a fluid
does not act like the rest of the fluid, but instead acts something like a stretched
membrane. This allows insects to walk on water, and water drops to hang from a
faucet. This phenomena is due to attractive forces between the fluid molecules
called surface tension. The surface tension is defined as the force per unit
length that acts along any line of a surface. Surface tension is defined by

γ = F/L

where γ is the Greek letter gamma, and is defined as the surface tension. Table
10-4 lists surface tensions for various fluids. When a small object is placed in a
pool of water it depresses the water and this surface tension force tends to hold
up the object. Look at the forces on the object.
θ
The vertical forces are given by γ = F/L
ΣF = 0
w - 2γLcosθ = 0
If the object is totally submerged, then L for each side is half of the
circumference, or L = π r.
Then,
w = 2γπr cosθ .

So for a spherical object to float on a liquid, its weight must be approximately


less than 2γπr , since the maximum value of cosθ is 1. If the weight is more than
that, then surface tension will not play a real role in the problem, and we just use
Archimedes’ principle and buoyancy to analyze the situation.

Surface tension also explains why water rises up slightly in a glass, or can even
climb up a very thin tube. This phenomena is called capillarity. Suppose we put
a thin tube vertically into a pool of liquid.

Using Newton’s second law, we look at the


forces on the water

γ L - mg = 0
γ(2πr) - ρ(πr2h)g = 0

11
h = 2γ /(ρgr)

If r gets very large, then h is very small. So the fluid will only rise any
appreciable height if the radius of the tube is very small.
5.1 Drag
When an object moves through a fluid it experiences a drag. When we move
through the ocean of air that we live in, we experience a drag that we call air
resistance. This drag, analogous to air resistance, is experienced by any object
moving through any fluid. The drag that an object experiences is characterized
by something we call the Reynolds number. The Reynold’s number is defined as

R = vL ρ /η

where v is the velocity of the object relative to the fluid, ρ is the density,η is the
coefficient of viscosity, and L is a characteristic length of the object which
depends on its shape. As long as the Reynold’s number is less than about one,
then the fluid flow around the object is laminar and the force of drag is
proportional to the velocity

F D ∝ v = kv.

That means that as long as the Reynold’s number is below one, when your speed
doubles, the force of drag acting against your direction of motion also doubles.
When the Reynold’s number is greater than one, the flow of air is no longer
laminar. Instead it is turbulent, and the drag is proportional to the square of the
velocity.

F D ∝ v2

This means when you velocity doubles, your drag is increased by a factor of
four. The more aerodynamic an object is the lower the Reynold’s number. We
try to make objects aerodynamic not only to decrease wind resistance, but because
if the Reynold’s number is below about 1, then the drag of an object will increase
linearly with velocity rather than quadratically. On a bicycle, the drag increases
approximately with the square of the velocity, so when your speed doubles, the
drag from wind resistance is increased by a factor of four.

Let us consider the case of Reynold’s number below one with the fluid flow
laminar. Then FD = kv, where k depends on the size and shape of the object and
on the viscosity of the fluid. For a spherical object, k = 6πrη, so the force of
drag becomes

12
FD = 6πrηv which is called Stoke’s Equation. B FD

Consider an object falling in a fluid. There are


three forces acting on it, gravity, buoyancy, and
drag. By Newton’s second law, mg

mg - B - F D = ma

So when the flow is laminar,

mg - B - kv = ma
ρ OVg - ρ FVg - kv = ma

where ρO is the density of the object, and ρF is the density of the fluid.
When the sum of the forces equal, and there is no further acceleration, then the
object reaches its terminal velocity. (Remember this is all for the case of laminar
flow)

ρ OVg - ρ FVg - kv T = 0

v T = (ρ O - ρ F)Vg/k

13
Lecture PowerPoints

Chapter 10
Physics: Principles with
Applications, 6th edition
Giancoli

© 2005 Pearson Prentice Hall


This work is protected by United States copyright laws and is provided solely for
the use of instructors in teaching their courses and assessing student learning.
Dissemination or sale of any part of this work (including on the World Wide Web)
will destroy the integrity of the work and is not permitted. The work and materials
from it should never be made available to students except by instructors using
the accompanying text in their classes. All recipients of this work are expected to
abide by these restrictions and to honor the intended pedagogical purposes and
the needs of other instructors who rely on these materials.
Chapter 10
Fluids
Units of Chapter 10
•Phases of Matter
•Density and Specific Gravity
•Pressure in Fluids
•Atmospheric Pressure and Gauge Pressure
•Pascal’s Principle
•Measurement of Pressure; Gauges and the
Barometer
•Buoyancy and Archimedes’ Principle
Units of Chapter 10
•Fluids in Motion; Flow Rate and the Equation of
Continuity
•Bernoulli’s Equation
•Applications of Bernoulli’s Principle: from
Torricelli to Airplanes, Baseballs, and TIA
•Viscosity
•Flow in Tubes: Poiseuille’s Equation, Blood Flow
•Surface Tension and Capillarity
•Pumps, and the Heart
10-1 Phases of Matter
The three common phases of matter are solid,
liquid, and gas.
A solid has a definite shape and size.
A liquid has a fixed volume but can be any
shape.
A gas can be any shape and also can be easily
compressed.
Liquids and gases both flow, and are called
fluids.
10-2 Density and Specific Gravity
The density ρ of an object is its mass per unit
volume:
(10-1)

The SI unit for density is kg/m3. Density is also


sometimes given in g/cm3; to convert g/cm3 to
kg/m3, multiply by 1000.
Water at 4°C has a density of 1 g/cm3 = 1000 kg/m3.
The specific gravity of a substance is the ratio of
its density to that of water.
10-3 Pressure in Fluids
Pressure is defined as the force per unit area.

Pressure is a scalar; the units of pressure in the


SI system are pascals:
1 Pa = 1 N/m2

Pressure is the same in every


direction in a fluid at a given
depth; if it were not, the fluid
would flow.
10-3 Pressure in Fluids

Also for a fluid at rest, there is no


component of force parallel to any
solid surface – once again, if there
were the fluid would flow.
10-3 Pressure in Fluids
The pressure at a depth h below the surface of
the liquid is due to the weight of the liquid above
it. We can quickly calculate:
(10-3)

This relation is valid


for any liquid whose
density does not
change with depth.
10-4 Atmospheric Pressure and Gauge
Pressure
At sea level the atmospheric pressure is about
; this is called one
atmosphere (atm).
Another unit of pressure is the bar:

Standard atmospheric pressure is just over 1 bar.

This pressure does not crush us, as our cells


maintain an internal pressure that balances it.
10-4 Atmospheric Pressure and Gauge
Pressure

Most pressure gauges measure the pressure


above the atmospheric pressure – this is called
the gauge pressure.
The absolute pressure is the sum of the
atmospheric pressure and the gauge pressure.
10-5 Pascal’s Principle
If an external pressure is applied to a confined
fluid, the pressure at every point within the fluid
increases by that amount.
This principle is used, for example, in hydraulic
lifts and hydraulic brakes.
10-6 Measurement of Pressure; Gauges and
the Barometer

There are a number of different types of


pressure gauges. This one is an open-
tube manometer. The pressure in the
open end is atmospheric pressure; the
pressure being measured will cause
the fluid to rise until
the pressures on both
sides at the same
height are equal.
10-6 Measurement of Pressure; Gauges and
the Barometer
Here are two more devices for
measuring pressure: the
aneroid gauge and the tire
pressure gauge.
10-6 Measurement of Pressure; Gauges and
the Barometer
This is a mercury barometer,
developed by Torricelli to
measure atmospheric pressure.
The height of the column of
mercury is such that the pressure
in the tube at the surface level is 1
atm.
Therefore, pressure is often
quoted in millimeters (or inches)
of mercury.
10-6 Measurement of Pressure; Gauges and
the Barometer
Any liquid can serve in a
Torricelli-style barometer,
but the most dense ones
are the most convenient.
This barometer uses water.
10-7 Buoyancy and Archimedes’ Principle
This is an object submerged in a fluid. There is a
net force on the object because the pressures at
the top and bottom of it are different.
The buoyant force is
found to be the upward
force on the same volume
of water:
10-7 Buoyancy and Archimedes’ Principle
The net force on the object is then the difference
between the buoyant force and the gravitational
force.
10-7 Buoyancy and Archimedes’ Principle

If the object’s density is less than that of water,


there will be an upward net force on it, and it will
rise until it is partially out of the water.
10-7 Buoyancy and Archimedes’ Principle
For a floating object, the fraction that is
submerged is given by the ratio of the object’s
density to that of the fluid.
10-7 Buoyancy and Archimedes’ Principle

This principle also works in


the air; this is why hot-air and
helium balloons rise.
10-8 Fluids in Motion; Flow Rate and the
Equation of Continuity
If the flow of a fluid is smooth, it is called streamline or
laminar flow (a).
Above a certain speed, the flow becomes turbulent (b).
Turbulent flow has eddies; the viscosity of the fluid is much
greater when eddies are present.
10-8 Fluids in Motion; Flow Rate and the
Equation of Continuity
We will deal with laminar flow.
The mass flow rate is the mass that passes a
given point per unit time. The flow rates at any
two points must be equal, as long as no fluid is
being added or taken away.
This gives us the equation of continuity:
(10-4a)
10-8 Fluids in Motion; Flow Rate and the
Equation of Continuity
If the density doesn’t change – typical for
liquids – this simplifies to .
Where the pipe is wider, the flow is slower.
10-9 Bernoulli’s Equation

A fluid can also change its


height. By looking at the
work done as it moves, we
find:

This is Bernoulli’s
equation. One thing it
tells us is that as the
speed goes up, the
pressure goes down.
10-10 Applications of Bernoulli’s
Principle: from Torricelli to Airplanes,
Baseballs, and TIA
Using Bernoulli’s principle, we find that the speed
of fluid coming from a spigot on an open tank is:

(10-6)
This is called
Torricelli’s theorem.
10-10 Applications of Bernoulli’s
Principle: from Torricelli to Airplanes,
Baseballs, and TIA
Lift on an airplane wing is due to the different
air speeds and pressures on the two surfaces
of the wing.
10-10 Applications of Bernoulli’s
Principle: from Torricelli to Airplanes,
Baseballs, and TIA

A sailboat can move against


the wind, using the pressure
differences on each side of
the sail, and using the keel to
keep from going sideways.
10-10 Applications of Bernoulli’s
Principle: from Torricelli to Airplanes,
Baseballs, and TIA

A ball’s path will curve due to its


spin, which results in the air
speeds on the two sides of the
ball not being equal.
10-10 Applications of Bernoulli’s
Principle: from Torricelli to Airplanes,
Baseballs, and TIA
A person with constricted
arteries will find that they
may experience a
temporary lack of blood to
the brain (TIA) as blood
speeds up to get past the
constriction, thereby
reducing the pressure.
10-10 Applications of Bernoulli’s
Principle: from Torricelli to Airplanes,
Baseballs, and TIA
A venturi meter can be used to measure fluid
flow by measuring pressure differences.
10-10 Applications of Bernoulli’s
Principle: from Torricelli to Airplanes,
Baseballs, and TIA
Air flow across the top helps smoke go up a
chimney, and air flow over multiple openings can
provide the needed circulation in underground
burrows.
10-11 Viscosity
Real fluids have some internal friction, called
viscosity.
The viscosity can be measured; it is found from
the relation
(10-8)

where η is the coefficient of viscosity.


10-12 Flow in Tubes; Poiseuille’s
Equation, Blood Flow

The rate of flow in a fluid in a round tube


depends on the viscosity of the fluid, the
pressure difference, and the dimensions of the
tube.
The volume flow rate is proportional to the
pressure difference, inversely proportional to
the length of the tube and to the pressure
difference, and proportional to the fourth power
of the radius of the tube.
10-12 Flow in Tubes; Poiseuille’s
Equation, Blood Flow
This has consequences for blood flow – if the
radius of the artery is half what it should be, the
pressure has to increase by a factor of 16 to
keep the same flow.
Usually the heart cannot work that hard, but
blood pressure goes up as it tries.
10-13 Surface Tension and Capillarity
The surface of a liquid at rest is not perfectly flat;
it curves either up or down at the walls of the
container. This is the result of surface tension,
which makes the surface behave somewhat
elastically.
10-13 Surface Tension and Capillarity

Soap and detergents lower the surface tension


of water. This allows the water to penetrate
materials more easily.
Water molecules are
more strongly
attracted to glass than
they are to each other;
just the opposite is
true for mercury.
10-13 Surface Tension and Capillarity
If a narrow tube is placed in a fluid, the fluid will
exhibit capillarity.
10-14 Pumps, and the Heart
This is a simple reciprocating pump. If it is to be
used as a vacuum pump, the vessel is connected
to the intake; if it is to be used as a pressure
pump, the vessel is connected to the outlet.
10-14 Pumps, and the Heart
(a) is a centrifugal pump; (b) a rotary oil-seal pump;
(c) a diffusion pump
10-14 Pumps, and the Heart
The heart of a human, or any other animal, also
operates as a pump.
10-14 Pumps, and the Heart
In order to measure blood pressure, a cuff is
inflated until blood flow stops. The cuff is then
deflated slowly until blood begins to flow
while the heart is pumping, and then
deflated some more until the blood
flows freely.
Summary of Chapter 10
• Phases of matter: solid, liquid, gas.
• Liquids and gases are called fluids.
• Density is mass per unit volume.
• Specific gravity is the ratio of the density of the
material to that of water.
• Pressure is force per unit area.
• Pressure at a depth h is ρgh.
• External pressure applied to a confined fluid is
transmitted throughout the fluid.
Summary of Chapter 10

• Atmospheric pressure is measured with a


barometer.
• Gauge pressure is the total pressure minus the
atmospheric pressure.
• An object submerged partly or wholly in a fluid
is buoyed up by a force equal to the weight of
the fluid it displaces.
• Fluid flow can be laminar or turbulent.
• The product of the cross-sectional area and the
speed is constant for horizontal flow.
Summary of Chapter 10

• Where the velocity of a fluid is high, the


pressure is low, and vice versa.
• Viscosity is an internal frictional force within
fluids.
• Liquid surfaces hold together as if under
tension.
Chapter 16 Temperature and Heat
• Temperature and the Zeroth Law of
Thermodynamics
• Temperature Scales
• Thermal Expansion
• Heat and Mechanical Work
• Specific Heats
• Conduction, Convection, and Radiation
16-1 Temperature and the Zeroth Law of
Thermodynamics
Definition of heat:
Heat is the energy transferred between objects
because of a temperature difference.
Objects are in thermal contact if heat can flow
between them.
When the transfer of heat between objects in
thermal contact ceases, they are in thermal
equilibrium.
16-1 Temperature and the Zeroth Law of
Thermodynamics
The zeroth law of
thermodynamics:
If object A is in thermal
equilibrium with object B, and
object C is also in thermal
equilibrium with object B,
then objects A and C will be in
thermal equilibrium if brought
into thermal contact.
That is, temperature is the only factor that
determines whether two objects in thermal contact
are in thermal equilibrium or not.
16-2 Temperature Scales
The Celsius scale and The Fahrenheit scale:
Water freezes at 0°Celsius = 32 °Fahrenheit.
Water boils at 100°Celsius = 212°Fahrenheit.
The plot of TF vs TC is a
straight line with
a slope of ∆TF /∆
∆TC =9/5
From 0 Celsius to 100 Celsius, temperature
increases 100 degree C,
From 32 °F to 212°F, temperature increases 180°F

Every 5 Celsius of temperature change is equal to


9 Fahrenheit of temperature change.
16-2 Temperature Scales
The pressure in a ideal gas is proportional to its
temperature. The proportionality constant (slope) is different
for different gases, but they all reach zero pressure at the
same temperature, which we call absolute zero:
The Kelvin scale
is similar to the
Celsius scale,
except that the
Kelvin scale has
its zero at
absolute zero.

SI unit: K, Kelvin
16-2 Temperature Scales
The three temperature scales compared:

Every 5 Celsius
of temperature
change is equal
to __ Fahrenheit
of temperature
change, and
is equal to __
Kelvin of
temperature
change.
16-3 Thermal Expansion

Most substances expand when heated; the


change in length or volume is typically
proportional to the change in temperature.
The proportionality constant is called the
coefficient of linear expansion.
16-3 Thermal Expansion
The expansion of an area of a flat substance is
derived from the linear expansion in both
directions:

Holes expand as well:


16-3 Thermal Expansion
The change in volume of a solid is also derived
from the linear expansion:

For liquids and gases, only the coefficient of


volume expansion is defined:

In general what expends more


due to same temperature
change? Liquid or Solid?
16-3 Thermal Expansion
Some typical coefficients of volume expansion:

Water also expands when it is


heated, except when it is close to
freezing; it actually expands a
little bit (0.01% )when cooling
from 4°C to 0°C.
16-4 Heat and Mechanical Work
Experimental work has shown that heat is
another form of energy.

James Joule used a


device similar to this
one to measure the
mechanical equivalent
of heat:
16-4 Heat and Mechanical Work
One kilocalorie (kcal) is defined as the amount
of heat needed to raise the temperature of 1 kg
of water from 14.5°C to 15.5°C.
16-5 Specific Heats

The heat capacity of an object is the amount of


heat added to it divided by its rise in
temperature:

Q is positive if ∆T is positive; that is, if heat is


added to a system.
Q is negative if ∆T is negative; that is, if heat is
removed from a system.
16-5 Specific Heats
The heat capacity of an object
depends on its mass.
A quantity which is a property
only of the material is the
specific heat:
16-5 Specific Heats
When a hot object is put in water, it and the water come to thermal
equilibrium. If the mass of the flask can be ignored, and the
insulation keeps any heat from escaping:
The final
temperatures of
the object and
the water will be
equal, Tf.

The total energy of the system is conserved.


Water raises T, so that Qw > 0.
Block lowers T, so that Qb > 0. Qw+Qb=0
mwcw(Tf-Tw) + mbcb(Tf-Tb) = 0 (Use T final minus T initial)
Qw+Qb=0
mwcw(Tf-Tw) + mbcb(Tf-Tb) = 0
(Pay attention to the sign of Q, Use T final minus T initial)

Which one of the following is incorrect?

mwcw(Tf-Tw) = mbcb(Tf-Tb)

mwcw(Tf-Tw) = mbcb(Tb-Tf)
16-6 Conduction, Convection, and Radiation

Conduction, convection, and radiation are


three ways that heat can be exchanged.
Conduction is the flow of heat directly through
a physical material.
16-6 Conduction, Convection, and Radiation
Experimentally, it is found that the amount of
heat Q that flows through a rod:
• increases proportionally to the cross-
sectional area A
• increases proportionally to the temperature
difference from one end to the other
• increases steadily with time
• decreases with the length of the rod
The constant k is
called the thermal
conductivity of the
rod.
16-6 Conduction, Convection, and Radiation

Some typical thermal


conductivities:

Substances with high


thermal conductivities
are good conductors of
heat; those with low
thermal conductivities
are good insulators.
16-6 Conduction, Convection, and Radiation
Convection is the flow of fluid
due to a difference in
temperatures, such as warm
air rising. The fluid “carries”
the heat with it as it moves.
16-6 Conduction, Convection, and Radiation
All objects give off energy in the form of radiation, as
electromagnetic waves – infrared, visible light, ultraviolet –
which, unlike conduction and convection, can transport heat
through a vacuum.
Objects that are hot enough will glow – first red, then yellow,
white, and blue. Objects at body temperature radiate in the
infrared, and can be seen with night vision binoculars.
16-6 Conduction, Convection, and Radiation
The amount of energy radiated by an object due to its
temperature is proportional to its surface area and also
to the fourth (!) power of its temperature.
It also depends on the emissivity, which is a number
between 0 and 1 that indicates how effective a radiator
the object is; a perfect radiator would have an
emissivity of 1.

Here, e is the emissivity, and σ is the Stefan-


Boltzmann constant:
Summary of Chapter 16
• Heat is the energy transferred between objects
due to a temperature difference.
• Objects are in thermal contact if heat can flow
between them.
• Objects that are in thermal contact without any
flow of heat are in thermal equilibrium.
• Thermodynamics is the study of physical
processes that involve heat.
• If objects A and B are both in thermal
equilibrium with C, they are in thermal
equilibrium with each other.
Summary of Chapter 16
• Temperature determines whether two
objects will be in thermal equilibrium.
• Celsius scale: water freezes at 0°C, boils at
100°C
• Fahrenheit: water freezes at 32°F, boils at
212°F
• The lowest attainable temperature is
absolute zero.
• Kelvin: absolute zero is 0 K; water freezes at
273.15 K and boils at 373.15 K
Summary of Chapter 16
• Temperature scale conversions:

• Most substances expand when heated.


• Linear expansion:
• Volume expansion:
• Water contracts when heated from 0°C to
4°C.
Summary of Chapter 16
• Heat is a form of energy:
• Heat capacity of an object:

• Specific heat is heat capacity per unit mass:

• Energy is conserved in heat flow.


Summary of Chapter 16
• Conduction: heat exchange from one part of a
material to a cooler part, with no bulk motion of
the material.
• Heat exchanged in time t:

• Convection is heat exchange due to the bulk


motion of an unevenly heated fluid.
• Radiation is heat exchange due to
electromagnetic radiation.
Summary of Chapter 16 Optional
• Radiated power as a function of temperature:

• Stefan-Boltzmann constant:
Lecture PowerPoints

Chapter 15
Physics: Principles with
Applications, 6th edition
Giancoli

© 2005 Pearson Prentice Hall


This work is protected by United States copyright laws and is provided solely for
the use of instructors in teaching their courses and assessing student learning.
Dissemination or sale of any part of this work (including on the World Wide Web)
will destroy the integrity of the work and is not permitted. The work and materials
from it should never be made available to students except by instructors using
the accompanying text in their classes. All recipients of this work are expected to
abide by these restrictions and to honor the intended pedagogical purposes and
the needs of other instructors who rely on these materials.
Chapter 15
The Laws of Thermodynamics
Units of Chapter 15

•The First Law of Thermodynamics


•Thermodynamic Processes and the First Law
•Human Metabolism and the First Law
•The Second Law of Thermodynamics –
Introduction
•Heat Engines
•Refrigerators, Air Conditioners, and Heat
Pumps
Units of Chapter 15
•Entropy and the Second Law of
Thermodynamics
•Order to Disorder
•Unavailability of Energy; Heat Death
•Evolution and Growth; “Time’s Arrow”
•Statistical Interpretation of Entropy and the
Second Law
•Thermal Pollution and Global Warming
15-1 The First Law of Thermodynamics
The change in internal energy of a closed
system will be equal to the energy added to the
system minus the work done by the system on
its surroundings.

(15-1)

This is the law of conservation of energy,


written in a form useful to systems involving
heat transfer.
15-2 Thermodynamic Processes and the
First Law

An isothermal process is
one where the temperature
does not change.
15-2 Thermodynamic Processes and the
First Law

In order for an isothermal process to take


place, we assume the system is in contact
with a heat reservoir.
In general, we assume that the system
remains in equilibrium throughout all
processes.
15-2 Thermodynamic Processes and the
First Law

An adiabatic process is one where there is no


heat flow into or out of the system.
15-2 Thermodynamic Processes and the
First Law
An isobaric process (a) occurs at constant
pressure; an isovolumetric one (b) at constant
volume.
15-2 Thermodynamic Processes and the
First Law

If the pressure is constant, the work done is the


pressure multiplied by the change in volume:

(15-3)

In an isometric process, the volume does not


change, so the work done is zero.
15-2 Thermodynamic Processes and the
First Law
For processes where the pressure varies, the
work done is the area under the P-V curve.
15-2 Thermodynamic Processes and the
First Law
15-3 Human Metabolism and the First Law
If we apply the first law of thermodynamics to
the human body:

we know that the body can do work. If the


internal energy is not to drop, there must be
energy coming in. It isn’t in the form of heat;
the body loses heat rather than absorbing it.
Rather, it is the chemical potential energy
stored in foods.
15-3 Human Metabolism and the First Law

The metabolic rate is the rate at which internal


energy is transformed in the body.
15-4 The Second Law of
Thermodynamics – Introduction

The absence of the process illustrated above


indicates that conservation of energy is not the
whole story. If it were, movies run backwards
would look perfectly normal to us!
15-4 The Second Law of
Thermodynamics – Introduction

The second law of thermodynamics is a


statement about which processes occur and
which do not. There are many ways to state the
second law; here is one:

Heat can flow spontaneously from a hot object


to a cold object; it will not flow spontaneously
from a cold object to a hot object.
15-5 Heat Engines
It is easy to produce thermal energy using
work, but how does one produce work using
thermal energy?

This is a heat engine;


mechanical energy can
be obtained from
thermal energy only
when heat can flow from
a higher temperature to
a lower temperature.
15-5 Heat Engines

We will discuss only engines that run in a


repeating cycle; the change in internal energy
over a cycle is zero, as the system returns to
its initial state.
The high temperature reservoir transfers an
amount of heat QH to the engine, where part of
it is transformed into work W and the rest, QL,
is exhausted to the lower temperature
reservoir. Note that all three of these quantities
are positive.
15-5 Heat Engines
A steam engine is one type of heat engine.
15-5 Heat Engines
The internal combustion engine is a type of heat
engine as well.
15-5 Heat Engines

Why does a heat engine need a temperature


difference?
Otherwise the work done on the system in one
part of the cycle will be equal to the work done
by the system in another part, and the net work
will be zero.
15-5 Heat Engines
The efficiency of the heat engine is the ratio of
the work done to the heat input:

Using conservation of energy to eliminate W,


we find:

(15-4a)

(15-4b)
15-5 Heat Engines
The Carnot engine was created to examine the
efficiency of a heat engine. It is idealized, as it
has no friction. Each leg of its cycle is reversible.
The Carnot cycle consists of:
• Isothermal expansion
• Adiabatic expansion
• Isothermal compression
• Adiabatic compression
An example is on the next slide.
15-5 Heat Engines
15-5 Heat Engines
For an ideal reversible engine, the efficiency can
be written in terms of the temperature:

(15-5)

From this we see that 100% efficiency can be


achieved only if the cold reservoir is at absolute
zero, which is impossible.
Real engines have some frictional losses; the
best achieve 60-80% of the Carnot value of
efficiency.
15-6 Refrigerators, Air Conditioners, and Heat
Pumps
These appliances can be thought of as heat
engines operating in reverse.

By doing work, heat is


extracted from the cold
reservoir and exhausted to
the hot reservoir.
15-6 Refrigerators, Air Conditioners, and Heat
Pumps
15-6 Refrigerators, Air Conditioners, and Heat
Pumps

Refrigerator performance is measured by the


coefficient of performance (COP):
(15-6a)

Substituting:
(15-6b)

(15-6c)
15-6 Refrigerators, Air Conditioners, and Heat
Pumps
A heat pump can heat a house in the winter:

(15-7)
15-7 Entropy and the Second Law of
Thermodynamics

Definition of the change in entropy S when


an amount of heat Q is added:

(15-8)

Another statement of the second law of


thermodynamics:
The total entropy of an isolated system never
decreases.
15-8 Order to Disorder

Entropy is a measure of the disorder of a


system. This gives us yet another statement of
the second law:
Natural processes tend to move toward a state
of greater disorder.

Example: If you put milk and sugar in your


coffee and stir it, you wind up with coffee that
is uniformly milky and sweet. No amount of
stirring will get the milk and sugar to come
back out of solution.
15-8 Order to Disorder

Another example: when a tornado hits a


building, there is major damage. You never see
a tornado approach a pile of rubble and leave a
building behind when it passes.
Thermal equilibrium is a similar process – the
uniform final state has more disorder than the
separate temperatures in the initial state.
15-9 Unavailability of Energy; Heat Death

Another consequence of the second law:


In any natural process, some energy becomes
unavailable to do useful work.

If we look at the universe as a whole, it seems


inevitable that, as more and more energy is
converted to unavailable forms, the ability to do
work anywhere will gradually vanish. This is
called the heat death of the universe.
15-10 Evolution and Growth; “Time’s Arrow”
Growth of an individual, and evolution of a
species, are both processes of increasing order.
Do they violate the second law of
thermodynamics?
No! These are not isolated systems. Energy
comes into them in the form of food, sunlight,
and air, and energy also leaves them.
The second law of thermodynamics is the one
that defines the arrow of time – processes will
occur that are not reversible, and movies that
run backward will look silly.
15-11 Statistical Interpretation of Entropy and
the Second Law

A macrostate of a system is specified by giving


its macroscopic properties – temperature,
pressure, and so on.
A microstate of a system describes the position
and velocity of every particle.
For every macrostate, there are one or more
microstates.
15-11 Statistical Interpretation of Entropy and
the Second Law

A simple example: tossing four coins. The


macrostates describe how many heads and tails
there are; the microstates list the different ways of
achieving that macrostate.
15-11 Statistical Interpretation of Entropy and
the Second Law
We assume that each microstate is equally
probable; the probability of each macrostate
then depends on how many microstates are in it.
The number of microstates quickly becomes
very large if we have even 100 coins instead of
four; the table on the next slide lists some
macrostates, how many microstates they have,
and the relative probability that each macrostate
will occur. Note that the probability of getting
fewer than 20 heads or tails is extremely small.
15-11 Statistical Interpretation of Entropy and
the Second Law
15-11 Statistical Interpretation of Entropy and
the Second Law
Now we can say that the second law does not
forbid certain processes; all microstates are
equally likely. However, some of them have an
extraordinarily low probability of occurring – a
lake freezing on a hot summer day, broken
crockery re-assembling itself; all the air in a room
moving into a single corner.
Remember how low some probabilities got just in
going from four coins to 100 – if we are dealing
with many moles of material, they can become so
rare as to be effectively impossible.
15-12 Thermal Pollution and Global Warming
The generation of electricity using solar energy
(a) does not involve a heat engine, but fossil-fuel
plants (b) and nuclear plants (c) do.
15-12 Thermal Pollution and Global Warming
The heat output of any heat
engine, QL, is referred to as
thermal pollution, as it must be
absorbed by the environment.
15-12 Thermal Pollution and Global Warming
Air pollution is also emitted by power plants,
industries, and consumers. Some of this
pollution results in a buildup of CO2 in the
atmosphere, contributing to global warming.
This can be minimized through careful choices
of fuels and processes.
The thermal pollution, however, is a
consequence of the second law, and is
unavoidable; it can be reduced only by reducing
the amount of energy we use.
15-12 Thermal Pollution and Global Warming

Problem solving:
1. Define what is part of the system and what is
part of the surroundings.
2. Be careful of the signs for work and heat
when using the first law of thermodynamics.
3. Be careful about units; be sure that you are
using the same ones throughout the problem.
4. Express temperatures in kelvins.
15-12 Thermal Pollution and Global Warming
5. Efficiency is always less than 1.
6. Entropy increases when heat is added, and
decreases when heat is removed.
Summary of Chapter 15
• First law of thermodynamics:
• Isothermal process: temperature is constant.
• Adiabatic process: no heat is exchanged.
• Work done by gas at constant pressure:

• Heat engine changes heat into useful work;


needs temperature difference.
•Efficiency of a heat engine:
Summary of Chapter 15
• Upper limit on efficiency:

• Refrigerators and air conditioners do work to


extract heat from a cooler region and send it to a
warmer region:

• A heat pump is similar:


Summary of Chapter 15
• Second law of thermodynamics:
• heat flows spontaneously from a hot object
to a cold one, but not the reverse
• a given amount of heat cannot be changed
entirely to work
• natural processes tend to increase entropy.
• Change in entropy:
• Entropy is a measure of disorder.
• As time goes on, less and less energy is available
to do useful work.
Chapter 17. Quantity of Heat
A PowerPoint Presentation by
Paul E. Tippens, Professor of Physics
Southern Polytechnic State University

© 2007
Photo © Vol. 05
Photodisk/Getty

FOUNDRY: It requires about 289 Joules of heat to


melt one gram of steel. In this chapter, we will
define the quantity of heat to raise the temperature
and to change the phase of a substance.
Objectives: After finishing this
unit, you should be able to:
• Define the quantity of heat in terms of the calorie,
the kilocalorie, the joule, and the Btu.

• Write and apply formulas for specific


heat capacity and solve for gains and
losses of heat.

• Write and apply formulas for


calculating the latent heats of fusion
and vaporization of various materials.
Heat Defined as Energy
Heat
Heat isis not
not something
something an an object
object has,
has, but
but rather
rather
energy
energy that
that itit absorbs
absorbs oror gives
gives up.
up. The
The heat
heat lost
lost by
by
the
the hot
hot coals
coals isis equal
equal to
to that
that gained
gained byby the
the water.
water.

Cool Thermal Equilibrium


water

Hot coals
Units of Heat
One calorie (1 cal) is the quantity of heat required
to raise the temperature of 1 g of water by 1 C0.

Example

10 calories of heat will


raise the temperature of
10 g of water by 10 C0.
Units of Heat (Cont.)
One kilocalorie (1 kcal) is the quantity
of heat required to raise the temperature
of 1 kg of water by 1 C0.

Example

10 kilocalories of heat will


raise the temperature of 10
kg of water by 10 C0.
Units of Heat (Cont.)
One British Thermal Unit (1 Btu) is the
quantity of heat required to raise the
temperature of 1 lb of water by 1 F0.

Example

10 Btu of heat will raise


the temperature of 10 lb of
water by 10 F0.
The Btu is an Outdated Unit
The British Thermal Unit (1 Btu) is discouraged, but
unfortunately remains in wide-spread use today. If it
is to be used, we must recognize that the pound unit
is actually a unit of mass, not weight.

When working with the Btu,


Btu we must
recall that the pound-mass is not a
variable quantity that depends on
gravity --
1 lb
one reason that the use of
the Btu is discouraged!
1 lb (1/32) slug
The SI Unit of Heat
Since
Since heat
heat isis energy,
energy, thethe joule
joule isis the
the preferred
preferred
unit.
unit. Then,
Then, mechanical
mechanical energy
energy and and heat
heat are
are
measured
measured in in the
the same
same fundamental
fundamental unit. unit.

Comparisons of Heat Units:


11 cal
cal == 4.186
4.186 JJ 11 Btu
Btu == 778
778 ftft lb
lb

11 kcal
kcal == 4186
4186 JJ 11 Btu
Btu == 252
252 cal
cal

11 Btu
Btu == 1055
1055 JJ
Temperature and Quantity of Heat
The effect of heat on temp- 200C 220C
erature depends on the
quantity of matter heated.

The same quantity of heat


is applied to each mass of 600 g
water in the figure.
200C 300C
The larger mass
experiences a smaller
increase in temperature.
200 g
Heat Capacity
The heat capacity of a substance is the heat
required to raise the temperature a unit degree.

Lead Glass Al Copper Iron


1000C 1000C 1000C 1000C 1000C

37 s 52 s 60 s 83 s 90 s

Heat capacities based on time to heat from zero


to 1000C. Which has the greatest heat capacity?
Heat Capacity (Continued)
All
All at 10000CC placed
at 100 placed on
on Paraffin
Paraffin Slab
Slab
Lead Glass Al Copper Iron

Iron and copper balls melt all the way through;


others have lesser heat capacities.
Specific Heat Capacity
The
The specific
specific heat
heat capacity
capacity of
of aa material
material isis the
the
quantity
quantity of
of heat
heat needed
needed to
to raise
raise the
the
temperature
temperature ofof aa unit
unit mass
mass through
through aa unit
unit
degree.
degree.
Q
c ; Q  mct
mt

Water:
Water: cc == 1.0 cal/g CC00 or
1.0 cal/g Btu/lb FF00 or
or 11 Btu/lb or 4186
4186 J/kg
J/kg K
K

Copper: c = 0.094 cal/g C00 or 390 J/kg K


Copper: c = 0.094 cal/g C or 390 J/kg K
Comparison of Heat Units: How much heat is
needed to raise 1-kg of water from 00 to 1000C?
The mass of one kg of water is:
1 kg = 1000 g = 0.454 lbm
Q  mct 1 lbm = 454 g
1 kg
For water: c = 1.0 cal/g C0
or 1 Btu/lb F0 or 4186 J/kg K

The heat required to do this job is:


10,000 cal 10 kcal
39.7 Btu 41, 860 J
Problem Solving Procedure
1. Read problem carefully and draw a rough sketch.
2. Make a list of all given quantities
3. Determine what is to be found.
4. Recall applicable law or formula and constants.
Q
c ; Q  mct
mt
Water: c = 1.0 cal/g C00or 1 Btu/lb F00 or 4186 J/kg K
Water: c = 1.0 cal/g C or 1 Btu/lb F or 4186 J/kg K

5. Determine what was to be found.


Example 1: A 500-g copper coffee mug
is filled with 200-g of coffee. How much
heat was required to heat cup and
coffee from 20 to 960C?
1. Draw sketch of problem.
problem
2. List given information.
Mug mass mm = 0.500 kg
Coffee mass mc = 0.200 kg
Initial temperature of coffee and mug: t0 = 200C
Final temperature of coffee and mug: tf = 960C
3. List what is to be found: Total heat to raise temp-
erature of coffee (water) and mug to 960C.
Example 1(Cont.): How much heat needed to
heat cup and coffee from 20 to 960C?
mm = 0.2 kg; mw = 0.5 kg.
4. Recall applicable formula or law:
Heat Gain or Loss: Q = mc t
5. Decide that TOTAL heat is that
required to raise temperature of mug
and water (coffee). Write equation.
QT = mmcm t + mwcw t
6. Look up specific Copper: cm = 390 J/kg C0
heats in tables: Coffee (water): cw = 4186 J/kg C0
Example 1(Cont.): How much heat needed
to heat cup and coffee from 20 to 960C?
mc = 0.2 kg; mw = 0.5 kg.
7. Substitute info and solve problem:
Copper: cm = 390 J/kg C0
Coffee (water): cw = 4186 J/kg C0
QT = mmcm t + mwcw t

Water: (0.20 kg)(4186 J/kgC0)(76 C0) t = 9600C - 2000C


t = 96 C - 20 C
0 0
Cup: (0.50 kg)(390 J/kgC )(76 C ) =
= 76
76 C
C00

QT = 63,600 J + 14,800 J Q
QTT == 78.4
78.4 kJ
kJ
A Word About Units
The substituted units must be consistent with those of
the chosen value of specific heat capacity.

For example: Water cw = 4186 J/kg C0 or 1 cal/g C0

Q = mwcw t

IfIfThe
you units
use for
4186
1
you use 4186 Q
cal/g , m,
J/kg
1 cal/gJ/kgC 00C
C C and
for
for c,t c,c,
00 for
for
c,
thenmust
then Qbe
Q consistent
must
must be
be in with and
in calories,
joules,
calories,
joules, and
m
and
m
andthose
must
m
must based
m must
be
be in
must inbeoninthe
kilograms.
be in value of
grams.
kilograms.
grams.
the constant c.
Conservation of Energy
Whenever there is a transfer of heat within a
system, the heat lost by the warmer bodies must
equal the heat gained by the cooler bodies:

 (Heat Losses) ==  (Heat


(Heat Losses) (Heat Gained)
Gained)

Cool Thermal Equilibrium


water
Hot
iron
Example 2: A handful of copper 100
shot is heated to 900C and then water
dropped into 80 g of water in an 0 shot
insulated cup at 10 C. If the
0 90
equilibrium temperature is 180C, Insulator
what was the mass of the copper?
cw = 4186 J/kg C0; cs = 390 J/kg C0 te= 180C

mw = 80 g; tw= 100C; ts = 900C


Heat lost by shot = heat gained by water
mscs(900C - 180C) = mwcw(180C - 100C)

Note: Temperature differences are [High - Low] to


insure absolute values (+) lost and gained.
Example 2: (Cont.)
ms = ? 100 180C
water
900 shot
Insulator 80 g of Water
Heat lost by shot = heat gained by water
mscs(900C - 180C) = mwcw(180C - 100C)
ms(390 J/kgC0)(72 C0) = (0.080 kg)(4186 J/kgC0)(8 C0)
2679 J
ms   0.0954 kg m
mss == 95.4
95.4 gg
28,080 J/kg
Change of Phase
When
When aa change
change ofof phase
phase occurs,
occurs, there
there isis only
only aa
change
change in
in potential
potential energy
energy of
of the
the molecules.
molecules. The The
temperature
temperature isis constant
constant during
during the
the change.
change.

Solid Liquid Vaporization


Gas
fusion

Q = mLf Q = mLv

Terms:
Terms: Fusion,
Fusion, vaporization,
vaporization, condensation,
condensation, latent
latent
heats,
heats, evaporation,
evaporation, freezing
freezing point,
point, melting
melting point.
point.
Change of Phase
The latent heat of fusion (Lf) of a substance is
Q
the heat per unit mass required to change the Lf 
substance from the solid to the liquid phase of m
its melting temperature.
For
ForWater:
Water: LLff==80
80cal/g
cal/g==333,000
333,000J/kg
J/kg

The latent heat of vaporization (Lv) of a


Q
substance is the heat per unit mass required Lv 
to change the substance from a liquid to a m
vapor at its boiling temperature.

For
ForWater:
Water: LLvv==540
540cal/g
cal/g==2,256,000
2,256,000 J/kg
J/kg
Melting a Cube of Copper
The heat Q required to melt a
substance at its melting temperature What Q
can be found if the mass and latent 2 kg to melt
heat of fusion are known. copper?

= mL
QQ = mLvv Lf = 134 kJ/kg
Example: To completely melt
2 kg of copper at 10400C, we need:
Q = mLf = (2 kg)(134,000 J/kg) Q
Q == 268
268 kJ
kJ
Example 3: How much heat is needed to
convert 10 g of ice at -200C to steam at 1000C?

First, let’s review the process graphically as shown:


temperature
t
ice steam 540 cal/g
100 C0

1 cal/gC0 steam
steam only
80 cal/g and
00C water water
ice
ciceand
= 0.5 cal/gC0
-20 C ice water
0 only
Q
Example 3 (Cont.): Step one is Q1 to convert
10 g of ice at -200C to ice at 00C (no water yet).

Q1 to raise ice to 00C: Q1 = mct


-200C 00C

t
1000C Q1 = (10 g)(0.5 cal/gC0)[0 - (-200C)]
Q1 = (10 g)(0.5 cal/gC0)(20 C0)

Q
Q11 == 100
100 cal
cal
00C
cice= 0.5 cal/gC0
-200C ice Q
Example 3 (Cont.): Step two is Q2 to convert
10 g of ice at 00C to water at 00C.

Q2 to melt 10 g of ice at 00C: Q2 = mLf


Melting
t
1000C
Q2 = (10 g)(80 cal/g) = 800 cal

Q
Q22 == 800
800 cal
cal

80 cal/g Add this to Q1 = 100 cal:


00C 900 cal used to this point.
ice and
-200C water Q
Example 3 (Cont.): Step three is Q3 to change
10 g of water at 00C to water at 1000C.

Q3 to raise water at 00C to 1000C.


00C to 1000C Q3 = mct ; cw= 1 cal/gC0
t Q3 = (10 g)(1 cal/gC0)(1000C - 00C)
1000C
Q
Q33 == 1000
1000 cal
cal
1 cal/gC0
Total = Q1 + Q2 + Q3
00C = 100 +900 + 1000
water
= 1900 cal
-200C
only
Q
Example 3 (Cont.): Step four is Q4 to convert
10 g of water to steam at 1000C? (Q4 = mLv)

Q4 to convert all water at 1000C


vaporization to steam at 1000C. (Q = mLv)

Q4 = (10 g)(540 cal/g) = 5400 cal


1000C

800 cal 5400 cal Total Heat:


1000
100 cal steam 7300
7300 cal
cal
cal
00C
ice and water and
-200C ice water only water Q
Example 4: How many grams of ice at 00C
must be mixed with four grams of steam in
order to produce water at 600C?
mi = ?
Ice must melt and then rise to 600C. ice
Steam must condense and drop to 600C. steam
Total Heat Gained = Total Heat Lost 4g
miLf + micwt = msLv + mscwt te = 600C

Note: All losses and gains are absolute values (positive).


Total Gained:
Total Gained: m
mi(80
(80 cal/g)
cal/g) +
+ m
m (1
(1 cal/gC
cal/gC00)(60
)(60 C
C00)- 00C )
i ii

Total
Lost: Lost:
(4 g)(540
(4 g)(540
cal/g) cal/g)
+ (4 g)(1
+ (4cal/gC 0)(1000)(40
g)(1 cal/gC C0 - 60
C00)C )
Example 4 (Continued)
Total Gained: mi(80 cal/g) + mi(1 cal/gC0)(60 C0)

Total Lost: (4 g)(540 cal/g) + (4 g)(1 cal/gC0)(40 C0)


mi = ?
Total Heat Gained = Total Heat Lost
80mi + 60mi = 2160 g +160 g 4g

2320 g te = 600C
mi  m
mi i == 16.6
16.6 gg
140
Example 5: Fifty grams of ice ice water
are mixed with 200 g of water
initially at 700C. Find the 00C 700C
equilibrium temperature of the 50 g 200 g
mixture.
Ice melts and rises to te te = ?
Water drops from 70 to te.

Heat Gained: miLf + micwt ; t = te - 00C

Gain = (50 g)(80 cal/g) + (50 g)(1 cal/gC0)(te - 00C )

Gain = 4000 cal + (50 cal/g)te


Example 5 (Cont.):

Gain = 4000 cal + (50 cal/g)te


00C 700C
50 g 200 g
Heat Lost = mwcwt
t = 700C - te [high - low] te = ?

Lost = (200 g)(1 cal/gC0)(700C- te )


Lost = 14,000 cal - (200 cal/C0) te

Heat Gained Must Equal the Heat Lost:


4000 cal + (50 cal/g)te = 14,000 cal - (200 cal/C0) te
Example 5 (Cont.):
Heat Gained Must Equal the Heat Lost:
4000 cal + (50 cal/g)te = 14,000 cal - (200 cal/C0) te
Simplifying, we have: (250 cal/C0) te = 10,000 cal

10,000 cal
te  0
 40 C
0
00C 700C
250 cal/C 50 g 200 g

ttee = 40 00C
= 40 C te = ?
Summary of Heat Units
One calorie (1 cal) is the quantity of heat
required to raise the temperature of 1 g of
water by 1 C0.
One kilocalorie (1 kcal) is the quantity of heat
required to raise the temperature of 1 kg of
water by 1 C0.

One British thermal unit (Btu) is the quantity


of heat required to raise the temperature of 1
lb of water by 1 F0.
Summary: Change of Phase
The latent heat of fusion (Lf) of a substance is
Q
the heat per unit mass required to change the Lf 
substance from the solid to the liquid phase of m
its melting temperature.
For
ForWater:
Water: LLff==80
80cal/g
cal/g==333,000
333,000J/kg
J/kg

The latent heat of vaporization (Lv) of a


Q
substance is the heat per unit mass required Lv 
to change the substance from a liquid to a m
vapor at its boiling temperature.

For
ForWater:
Water: LLvv==540
540cal/g
cal/g==2,256,000
2,256,000 J/kg
J/kg
Summary: Specific Heat Capacity

The
The specific
specific heat
heat capacity
capacity of of aa
material
material isis the
the quantity
quantity of
of heat
heat toto
raise
raise the
the temperature
temperature of of aa unit
unit
mass
mass through
through aa unit
unit degree.
degree.

Q
c ; Q  mct
mt
Summary: Conservation of Energy

Whenever there is a transfer of heat


within a system, the heat lost by the
warmer bodies must equal the heat
gained by the cooler bodies:

 (Heat Losses) ==  (Heat


(Heat Losses) (Heat Gained)
Gained)
Summary of Formulas:
Q
c ; Q  mct
mt
 (Heat Losses) ==  (Heat
(Heat Losses) (Heat Gained)
Gained)

Q
Lf  ; Q  mL f
m
Q
Lv  ; Q  mLv
m
CONCLUSION: Chapter 17
Quantity of Heat
SOLVED PROBLEMS OF THE SECOND LAW OF THERMODYNAMICS

1. A Carnot engine operates between 1900 C and 600 C. (a) What is the efficiency of this
Carnot engine? (b) If the efficiency is to be raised to 35%, to what value must the higher
temperature be raised if the lower temperature is kept constant?
Given: TL = 600 C = 273.15 + 60 = 333.15 K
TH = 1900 C = 273.15 + 190 = 463.15 K
𝑇𝐻 −𝑇𝐿 𝑇𝐿 333.15 𝐾
a) e = = 1 - = 1 - 463.15 𝐾 = 1 – 0.72 = 0.28
𝑇𝐻 𝑇𝐻
e = 28 %
b) e = 35%
𝑇𝐿 333.15 𝐾
0.35 = 1 - =1-
𝑇𝐻 𝑇𝐻
333.5 𝐾 − 333.15 𝐾
0.35 – 1 = -
𝑇𝐻
- 0.65TH = - 333.15 K TH =
− 0.65
TH = 512.54 K
2. A 1-kW refrigerator whose coefficient of performance(COP) is 2.0 takes heat
from a freezer compartment at -200 C and exhausts it at 400 C. (a) What is the
ideal COP at these temperatures? (b) How does the real COP of this refrigerator
compare with that of an ideal refrigerator? (c) At what rate does the refrigerator
remove heat from the freezer compartment?
Given : TL = -200 C = 253.15 K , TH = 400 C = 313.15 K, P = 1 kW = 1000 W

𝑇𝐿 253.15 𝐾
a) COP = = = 4.2
𝑇𝐻 −𝑇𝐿 313.15 𝐾 −253.15 𝐾

𝐶𝑂𝑃 𝑜𝑓 𝑡ℎ𝑒 𝑟𝑒𝑎𝑙 𝑟𝑒𝑓𝑟𝑖𝑔𝑒𝑟𝑎𝑡𝑜𝑟 2


b) = = 0.48
𝐶𝑂𝑃 𝑜𝑓 𝑡ℎ𝑒 𝑖𝑑𝑒𝑎𝑙 𝑟𝑒𝑓𝑟𝑖𝑔𝑒𝑟𝑎𝑡𝑜𝑟 4.2
Hence, the actual refrigerator is 48% as efficient as the ideal refrigerator.
𝑄𝐿 𝑄𝐿 𝑊 𝑊
c) COP = QL = COP(W) = COP( ) where P =
𝑊 𝑡 𝑡 𝑡
𝑄𝐿
= 2(1000 W) = 2000 W or 2 KW This means that for every
𝑡
1000 J of energy supplied to the refrigerator, it removes 2000 J of heat
from the refrigerator.
TEMPERATURE AND
HEAT
TEMPERATURE AND THERMAL
EQUILIBRIUM
TEMPERATURE
Temperature is often described as a measure of the hotness and coldness of an
object. A body that feels hot usually has a higher temperature than a similar body that
feels cold.
Many measurable properties of matter depend on temperature: the length of a
metal rod, steam pressure in a boiler, the ability of a wire to conduct an electric current,
the color of a very hot glowing object, etc.
Temperature is also related to the kinetic energies of the molecules of a material
or substance. Many scientists consider temperature as the average kinetic energy of all
the molecules of a substance. In this topic we’ll develop a macroscopic definition of
temperature.(The term macroscopic refers to the bulk properties of a substance like
pressure, volume, etc.)
Temperature is a property of a body or region of space that determines whether
or not there will be a net flow of heat into it or out of it from a neighboring body or
region and in which direction(if any) the heat will flow. (Dictionary of Physics)
THERMAL EXPANSION
When their temperatures are increased, most materials expand. Heat energy
added to a material causes the particles that make up the material to speed up. As the
speeds of the particles increase, the pressure also increases which results in the
expansion of the material due to the increase in internal force. What happens if there is
a loss of heat energy? It causes the material to shrink (or contract) in size.
Linear Expansion
It is observed that a rod changes its length when there is a temperature change.
The change in length of the rod also depends on the type of the rod. The change in
length is given by the equation:

ΔL = αLoΔT

where α = the coefficient of linear expansion


Lo = the original length
L = the new length = Lo + ΔL
ΔT = Tf – Ti where the unit of ΔT is C0 or F0
The value of ΔT is the same for C0 and K. For example, Ti = 500C = 323.15 K
and Tf = 1500C = 423.15 K; ΔT = Tf – Ti = 100 C0 = 100 K.
COEFFICIENTS OF EXPANSION
Sample Problems:
1. A steel-arch bridge in West Virginia is 518-m long. What will be its new
length if the temperature changes from -100C to 300C?
Given: Lo = 518 m, Tf = 300C, Ti = -100C, L=?
ΔL = αLoΔT = 1.2 X 10-5/C0 (518 m)(300C – (-100C))
= 1.2 X 10-5/C0 (518 m)(40C0)
ΔL = 0.24865 m
L = Lo + ΔL = 518 m + 0.24865 m
L = 518.24865 m
2. A steel ring with a 2.5000-in inside diameter at 200C is to be warmed and
slipped over a brass shaft with a 2.5020-in outside diameter at 200C. (a) To what
temperature should the ring be warmed? (b) If the ring and the shaft together are
cooled by some means such as liquid air, at what temperature will the ring just slip
off the shaft? (University Physics, 12th ed, #17.91, p. 606)
Given: do(s) = 2.5000 in, Ti = 200C
do(b) = 2.5020 in, Ti = 200C
Tf = ?
a) The change in length of the ring is (Δd) = 2.5020 in – 2.5000 in = 0.0020
in or 2.0 X 10-3 in.

Δd = αsdo(s)ΔT
2.0 X 10-3 in = 1.2 X 10-5/C0(2.5000 in)(Tf – 200C)
2.0 X 10-3 in = (3.0 X 10-5 in/C0)Tf – 6.0 X 10-4 in
- (3.0 X 10-5 in/C0)Tf = - 6.0 X 10-4 in - 2.0 X 10-3 in
− 2.6 𝑋 10−3 𝑖𝑛
Tf =
− 3.0 𝑋 10−5 𝑖𝑛/0𝐶
Tf = 86.70C
b) Original length of steel ring + change in length of steel ring =
original length of brass + change in length of brass shaft
do(s) + Δd(s) = do(b) + Δd(b)
do(s) + αdo(s) ΔT = do(b) + αbΔdo(b)
2.5000 in + 1.2 X 10-5/C0 (2.5000 in)(Tf – 200C ) =
2.5020 in + 2.0 X 10-5/C0(2.5020 in)(Tf – 200C)
2.5000 in + (3.0 X 10-5 in/0C) Tf – 6 X 10-4 in =
2.5020 in + (5.004 X 10-5 in/0C)Tf – 1.0008 X 10-3 in
(3.5 X 10-5 in/0C) Tf - (5.004 X 10-5 in/0C)Tf = 2.5020 in – 1.0008 X 10-3 in -
2.5000 in + 6 X 10-4 in
(- 2.004 X 10-5 in/0C) Tf = 1.5992 X 10-3 in
1.5992 𝑋 10−3 𝑖𝑛
Tf =
− 2.004 𝑋 10−5 𝑖𝑛/0𝐶
Tf = -79.80C
AREA EXPANSION
When a sheet of metal increases its temperature by the absorption of
heat, its dimension (specifically the area) expands. Its expansion is given
the formula of its area expansion
ΔA = γAoΔT
where γ = the coefficient of area expansion
Ao= the original area ΔaΔb
ΔT = Tf –Ti
Δa

How do we show that ΔA = γAoΔT = 2αAoΔT?


a
Let Ao = ab and A = Ao + ΔA
A = (a + Δa)(b + Δb ) = ab + 2aΔb + ΔaΔb b
neglect
A = Ao + ΔA = ab + 2aΔb + ΔaΔb Δb
Ao + ΔA = Ao + 2aαbΔT
Therefore, ΔA = 2αAoΔT where 2α = γ and ΔaΔb is a very small value and hence negligible.
Heating an Object with a Hole

If a sheet of metal has a whole in it, what happens to the


diameter of a hole when the temperature of the metal increases? A
common expectation is that as the temperature increases, the hole of
the material shrinks because the material shrinks into the hole. This
expectation is a misconception because as observed every linear
dimension of a material changes as the temperature changes;
therefore, as the material undergoes thermal expansion, any holes in
the material expand as well.
VOLUME EXPANSION

Observations showed that if the temperature change is not too


large (less than 1000C or so), the change in volume ΔV is
approximately proportional to the initial volume Vo and the
temperature changes ΔT:
ΔV = βVoΔT
where β = the coefficient of volume expansion
For solids, β = 3α. To derive this relationship, consider a cube
with a side length Lo and an initial volume Vo = Lo3 and V = L3.
From dV = (dV/dL)dL = 3L2dL. Replacing L and V by the
initial values Lo and Vo, dL = αLodT and dV = 3Lo2αLodT = 3αLo3dT.
Hence, dV = 3αVodT where β = 3α.
Sometimes the derivation makes use of the new volume
expressed as V = (Lo + ΔLo)3.
neglect neglect
V = (Lo + ΔLo)3 = Lo3 + 3Lo2ΔLo +
3LoΔLo2 + ΔLo3
V = Lo3 + 3Lo2ΔLo
Vo + ΔV = Vo + 3Lo2αLoΔT
ΔV = 3αLo3ΔT = 3αVoΔT
Example: A hollow aluminum cylinder 20.0 cm deep has an internal
capacity of 2.0 L at 200C. It is completely filled with turpentine, and
then warmed to 800C. How much turpentine overflows in cm3? (1 L =
1000 cm3, βturp = 9 X 10-4/0C, βAl= 3αAl = 0.72 X 10-4/0C)
Vover = ΔVturp - ΔVAl
= (βVo ΔT)turp – (βVo ΔT)Al
= 0.994 L = 994 cm3
EXERCISE
1. A small plastic container, called the coolant reservoir, catches the radiator
fluid that overflows when an automobile engine becomes hot. The radiator
is made of copper, and the coolant has a volume expansion of βcoolant = 4.10
X 10-4/C0. If the radiator is filled to its 15-quart capacity when the engine is
cold (6.00C), how much overflow from the radiator will spill into the
reservoir when the coolant reaches its operating temperature of 920C? αCu =
17 X 10-6/C0) (Physics by Cutnell and Johnson, 6th ed.) Ans. = 0.46 quart
2. A structural I-beam is 15.0-m long when installed at 20.00C. (a) How much
does its length change over the temperature extremes -30.00C to 50.00C?
(b) What is its new length? (Physics for Scientists and Engineers by Raymond
A. Serway, 4th ed.) Ans. (a) ΔL = 0.0132 m or 1.32 X 10-2 m (b) L = 15.0312 m
METHODS OF HEAT TANSFER
The three methods of heat transfer are (a) conduction, (b) convection, and (c)
radiation.
Conduction is a process of heat transfer by means of the collisions of the adjacent
molecules of a substance or a material. This process occurs because of temperature gradient
(dT/dx), that is, the temperature difference over that of the length of the material. In other
words, conduction occurs only when the body’s temperature is not uniform.
The rate of flow of heat, or the heat current (H), by conduction is observed to depend
𝑇 −𝑇𝐿
on the cross sectional area (A) of the material, the temperature gradient (ΔT/ΔL = 𝐻 ), and
𝐿
the type of material as represented by its thermal conductivity (k). The relationship is shown
by the formula,
𝑇𝐻 −𝑇𝐿 𝑄
H = kA . H=
𝐿 𝑡
A
TH
TL
H has a unit of J/s or watt (W). H

L
CONDUCTION THROUGH COMPOSITE MATERIALS

TH K1 TX K2 TL
H
L1 L2

𝑇𝐻 −𝑇𝑋 𝑇𝑋 −𝑇𝐿
H1 = k1A1 ( ) H2 = k2A2 ( )
𝐿1 𝐿2
In a steady-state condition, the heat current per unit area is the same all along the
thicknesses or the same in all layers, and there is no net heat gain or loss of energy in any
layer; no heat is accumulating in the wall because the inner and outer temperatures are
constant. In other words, the temperatures and the rate of heat transfer are constant in
time. Hence,
H = H1 =H2 =…… = HN
Since there are only two layers, H1 = H2. Therefore,
𝑇𝐻 −𝑇𝑥 𝑇𝑋 −𝑇𝐿
k1A1 ( ) = k2A2 ( )
𝐿1 𝐿2
Solving for Tx (the interface temperature):
𝑘1𝐴1𝑇𝐻 𝑘1𝐴1 𝑇𝑥 𝑘2𝐴2𝑇𝑥 𝑘2𝐴2𝑇𝐿
- = -
𝐿1 𝐿1 𝐿2 𝐿2
𝑘 𝐴 𝑇𝑥 𝑘 𝐴 𝑇 𝑘𝐴𝑇 𝑘𝐴𝑇
- 1 1 - 2 2 𝑥=- 1 1 𝐻- 2 2 𝐿
𝐿1 𝐿2 𝐿1 𝐿2
𝑘1𝐴1 𝑘2𝐴2 𝑘1𝐴1𝑇𝐻 𝑘2𝐴2𝑇𝐿
Tx ( + )= +
𝐿1 𝐿2 𝐿1 𝐿2

𝑘1𝐴1𝑇𝐻 𝑘2𝐴2𝑇𝐿 𝑘1𝐴1 𝑘2𝐴2


Tx = ( + )/ ( + )
𝐿1 𝐿2 𝐿1 𝐿2
When this last equation is substituted into Segway either H1 or H2, we get
𝐴1(𝑇𝐻 −𝑇𝐿) 𝐴2(𝑇𝐻 −𝑇𝐿)
H1 = 𝐿1 𝐿2 and H2 = 𝐿1 𝐿2
+ +
𝑘1 𝑘2 𝑘1 𝑘2
But L/k = R = thermal resistance, hence,
𝐴1(𝑇𝐻 −𝑇𝐿) 𝐴2(𝑇𝐻 −𝑇𝐿)
H1 = and H2 =
𝑅1+𝑅2 𝑅1+𝑅2
𝐴(𝑇𝐻 −𝑇𝐿)
H=
Σ𝑅
For the R-value: the lower the thermal conductivity, the higher is the R-value. Good heat
insulators have high R-value.
For composite materials arranged in parallel:

k1
TH
H TL
k2

H = H1 + H2+ …. + HN
SAMPLE PROBLEMS
1. A silver rod with a cross section of 10 cm2 and a length of 10 cm is in contact
with boiling water (1000C) at one end and ice (00C) at another end. What is the
rate of heat transfer along the rod?
Given: A = 10 cm2 = 0.001 m2 , L = 10 cm = 0.1 m, TH = 1000C , TL = 00C
2 (1000𝐶 −00𝐶)
H = (406 W/m.K)(0.001 m)
0.1 𝑚
H = 406 W
2. A carpenter builds an exterior house wall with a layer of wood 3.0 cm thick on
the outside and a layer of Styrofoam insulation 2.2 cm thick on the inside wall
surface. The wood has k = 0.080 W/m.K, and the Styrofoam has k = 0.01 W/m.K.
The interior surface temperature is 19.00C, and the exterior surface
temperature is – 10.00C. (a) What is the temperature at the plane where the
wood meets the Styrofoam? (b) What is the rate of heat flow per square meter
through this wall? (University Physics, 12th ed., Young et. al., # 17.67)
(a) Tx = - 5.80C (b) H/A = 11 W/m2

You might also like